You are on page 1of 56

129

5
Laws of Motion

ARISTOTLES FALLACY The SI unit of force is newton. (One newton force is that much
According to Aristotelian law an external force is required to keep force which produces an acceleration of 1ms2 in a body of mass
a body in motion. However an external force is required to 1 kg.
overcome the frictional forces in case of solids and viscous forces The CGS unit of force is dyne. (1N = 105 dyne)
in fluids which are always present in nature. The gravitational unit of force is kg-wt (kg-f) or g-wt (g-f)
1 kg-wt (kg-f) = 9.8 N, 1 g-wt (g-f) = 980dyne
LINEAR MOMENTUM (p)
Linear momentum of a body is the quantity of motion contained
r r Third law : To every action there is an equal and opposite
in the body. Momentum p = mv reaction. For example walking , swimming , a horse pulling a
It is a vector quantity having the same direction as the direction cart etc.
of the velocity. Its SI unit is kg ms1. r r
FAB = FBA
NEWTONS LAWS OF MOTION
Action and reaction act on different bodies and hence cannot
First law : A body continues to be in a state of rest or of uniform balance each other. Action and reaction occur simultaneously.
motion, unless it is acted upon by some external force to change Forces always occur in pairs.
its state.
Newtons first law gives the qualitative definition of force according EQUILIBRIUM OF A PARTICLE
to which force is that external cause which tends to change or A body is said to be in equilibrium when no net force acts on the
actually changes the state of rest or motion of a body. body.
r
Newtons first law of motion is the same as law of inertia given by i.e., SF = 0
Galileo.
Inertia is the inherent property of all bodies because of which Then SFx = 0, SFy = 0 and SFz = 0
they cannot change their state of rest or of uniform motion unless Stable equilibrium : If a body is slightly displaced from equilbrium
acted upon by an external force. position, it has the tendency to regain its original position, it is
Second law : The rate of change of momentum of a body is directly said to be in stable equilibrium.
proportional to the external force applied on it and the change
takes place in the direction of force applied. d 2u
In this case, P.E. is minimum. 2 = +ve
r dpr mdvr r dr



i.e., F = = = ma
dt dt So, the centre of gravity is lowest.
This is the equation of motion of constant mass system. For Unstable equilibrium : If a body, after being displaced from the
variable mass system such as rocket propulsion equilibrium position, moves in the direction of displacement, it is
r d (mvr ) said to be in unstable equilibrium.
F=
dt d 2u
In this case, P.E. is maximum. 2 = -ve
r m(dvr ) r dm dr
+v
And, F =
dt dt So, the centre of gravity is highest.
130
Neutral equilibrium : If a body, after being slightly displaced
from the equilibrium position has no tendency to come back or to d 2u
In this case, P.E. is constant 2 = constant
move in the direction of displacement the equilibrium is known to dr
be neutral.
The centre of gravity remains at constant height.

5.1
Solve following problems with the help of above text and (a) first law (b) second law
examples : (c) third law (d) All of the above
6. A cannon after firing recoils due to
1. Swimming is possible on account of
(a) conservation of energy
(a) Newtons first law of motion
(b) backward thrust of gases produced
(b) Newtons second law of motion
(c) Newtons third law of motion
(c) Newtons third law of motion
(d) Newtons first law of motion
(d) Newtons law of gravitation
7. Newtons second law measures the
2. Inertia is that property of a body by virtue of which the
(a) acceleration (b) force
body is
(c) momentum (d) angular momentum
(a) unable to change by itself the state of rest
8. We can derive Newtons
(b) unable to change by itself the state of uniform motion
(a) second and third laws from the first law
(c) unable to change by itself the direction of motion
(b) first and second laws from the third law
(d) All of the above
(c) third and first laws from the second law
3. An object will continue moving uniformly when
(d) All the three laws are independent of each other
(a) the resultant force on it is increasing continuously
9. A jet plane moves up in air because
(b) the resultant force is at right angles to its rotation
(a) the gravity does not act on bodies moving with high
(c) the resultant force on it is zero
speeds
(d) the resultant force on it begins to decrease
(b) the thrust of the jet compensates for the force of
4. A man getting down a running bus falls forward because
gravity
(a) of inertia of rest, road is left behind and man reaches
(c) the flow of air around the wings causes an upward
forward
force, which compensates for the force of gravity
(b) of inertia of motion upper part of body continues to
(d) the weight of air whose volume is equal to the volume
be in motion in forward direction while feet come to
of the plane is more than the weight of the plane
rest as soon as they touch the road.
10. When a body is stationary
(c) he leans forward as a matter of habit
(a) there is no force acting on it
(d) of the combined effect of all the three factors stated
(b) the force acting on it is not in contact with it
in (a), (b) and (c).
(c) the combination of forces acting on it balances each
5. A man is at rest in the middle of a pond of perfectly smooth
other
ice. He can get himself to the shore by making use of
(d) the body is in vacuum
Newtons

ANSWER KEY
1. (c) 2. (d) 3. (c) 4. (b) 5. (c) 6. (c) 7. (b) 8. (c) 9.(b) 10.(c)

COMMON FORCES IN MECHANICS


1. Weight : It is the force with which the earth attracts a body Case 1 Case 2
and is called force of gravity, For a body of mass m, where 2T
T
acceleration due to gravity is g, the weight 2T
W = mg T Massless
2. Tension : The force exerted by the ends of a loaded/stretched T T pulley
string (or chain) is called tension. The tension has a sense T T
of pull at its ends. m1 m2
m1g m2g
131
Case 3 4. Spring force : If an object is connected by spring and spring
is stretched or compressed by a distance x, then restoring
T T a
T' T' T1 force on the object F = kx
T
T m where k is a spring contact on force constant.
T
T1 T = ma 5. Frictional force : It is a force which opposes relative motion
If m = 0, T1 = T between the surfaces in contact. f = mN
i.e tension is same This will be discussed in detail in later section.
The tension in a string remains the same throughout the string if 6. Pseudo force : If a body of mass m is placed in a non-inertial
(a) string is massless, frame having aceleration ar , then it experiences a Pseudo
(b) pulley is massless or pulley is frictionless force acting in a direction opposite to the direction of ar .
Case 4 : String having mass r r
Fpseudo = ma
Negative sign shows that the pseudo force is always directed
in a direction opposite to the direction of the acceleration of
the frame.
y

Let the total mass of the string be M and length be L. Then mass
M a
per unit length is
L Fpseudo
Let x be the distance of the string from the mass m. Then the mass
m x

M z
of the shaded portion of string is x
L CONSTRAINT MOTION :
If the string is at rest then the tension T has to balance the wt of When the motion of one body is dependent on the other body, the
shaded portion of string and weight of mass m. relationship of displacements, velocities and accelerations of
the two bodies are called constraint relationships.
M
\T = m + x g Case 1 Pulley string system :
L
as x increases, the tension increases. Thus tension is non- X
uniform in a string having mass. F
3. Normal force : It measures how strongly one body presses
the other body in contact. It acts normal to the surface of x Block
contact.
Step 1 : Find the distance of the two bodies from fixed points.
mg Step 2 : The length of the string remain constant. (We use of
Case 1 N = mg
N this condition)
Therefore X + (X x) = constant 2X x = constant
Case 2
dX dx dX dx
a N mg = ma 2 =0 2 =
dt dt dt dt
m mg N = m(g + a)
dX
N 2Vp = v B Q = Vp = velocity of pulley
dt

dx
Case 3 = v B = velocity of block
dt
q N
mg sinq mg cos q Again differentiating we get, 2ap = aB
mg
dVp dvB
N = mg cos q a p = dt and a B = dt

q ap = acceleration of pulley, aB = acceleration of block
132
2 2 Case 1 : Masses M1 and M2 are tied to a string, which goes
Case 2 Here h + x + y = constt. On differentiating w.r.t t over a frictionless pulley
(a) If M2 > M1 and they move with acceleration a

h
1 q 2 T
F a T
x M1
a
[Negative sign with dy/dt shows that with increase in time, y
decreases] M1g M2
1 2x
dx dy
- = 0 cos q (v1 v2) = 0
2
2 h +x dt dt
2 M2g
FBD of M1, FBD of M2
x
Q cos q = T T
h 2 + x 2
Case 3 Wedge block system : Thin lines represents the condition M1 M2 a
a
of wedge block at t = 0 and dotted lines at t = t

M1g M2g
c
T - M1g = M1a M 2g - T = M 2a
ax
where T is the tension in the string. It gives
ay ay M 2 - M1 2M 1 M 2
a= g and T = g
M1 + M 2 M1 + M 2
q
B ax A (b) If the pulley begins to move with acceleration f,
Ax Ax downwards
q
uur M - M uur uur ur 2 M M uur uur
a = 2 1 ( g - f ) and 1 2
Ax = acceleration of wedge towards left T = (g - f )
M1 + M 2 M1 + M 2
ax, ay = acceleration of block as shown
Case 2 : Three masses M1, M2 and M3 are connected with
ay strings as shown in the figure and lie on a frictionless surface.
From D ABC , tan q = They are pulled with a force F attached to M1.
ax + Ax
T2 T2 T1 T 1
Frame of Reference : M3 M2 M1 F
Reference frames are co-ordinate systems in which an event is
The forces on M2 and M3 are as follows
described.
There are two types of reference frames M 2 + M3 M3
T1 = F and T2 = F;
(a) Inertial frame of reference: These are frames of reference M1 + M 2 + M 3 M1 + M 2 + M 3
in which Newtons laws hold good. These frames are at rest
with each other or which are moving with uniform speed F
Acceleration of the system is a =
with respect to each other. M1 + M 2 + M 3
All reference frames present on surface of Earth are Case 3 : Two blocks of masses M1 and M2 are suspended
supposed to be inertial frame of reference. vertically from a rigid support with the help of strings as shown
(b) Non inertial frame of reference: Newtons law do not in the figure. The mass M2 is pulled down with a force F.
hold good in non-inertial reference frame.
All accelerated and rotatory reference frames are non
inertial frame of reference. Earth is a non-intertial frame. T1
T1
When the observer is in non-inertial reference frame
a pseudo force is applied on the body under observation. M1 g M1
T2
Free Body Diagram (FBD) :
T2
Free body diagram of a mass is a separate diagram of that mass.
All forces acting on the mass are sketched. A FBD is drawn to M2g
M2
visualise the direct forces acting on a body.
F
133
The tension between the masses M1 and M2 will be (ii) When the mass M 1 moves downwards with
T2 = F + M2g acceleration a.
Tension between the support and the mass M1 will be Equation of motion for M1 and M2,
T1 = F + (M1 + M2)g
Case 4 : Two masses M1 and M2 are attached to a string which M1g sin q T = M1a ...(1)
passes over a pulley attached to the edge of a horizontal table. T M2g = M2a ...(2)
The mass M1 lies on the frictionless surface of the table. Solving eqns. (1) and (2) we get,
T
M sin q - M 2 M 2 M1 g
M1 a= 1 g; T =
M1 + M 2 M1 + M 2 (1 + sin q)
a
T
(a) If (M2/M1 = sinq) then the system does not accelerate.
(b) Changing position of masses, does not affect the
M2 tension. Also, the acceleration of the system remains
M2 g
unchanged.
(c) If M1 = M2 = M (say), then
Let the tension in the string be T and the acceleration of the 2 2
system be a. Then q q g q q Mg
a = cos - sin ; T = cos + sin
T = M1a ...(1) 2 2 2 2 2 2
M2g T = M2a ...(2)
Adding eqns. (1) and (2), we get Case 6 : Two masses M1 and M2 are attached to the ends of a
string over a pulley attached to the top of a double inclined
M2 M1 M 2
a= g and T = g plane of angle of inclination a and b.
M1 + M 2 M1 + M 2 Let M2 move downwards with acceleration a and the tension in
Case 5 : Two masses M1 and M2 are attached to the ends of a the string be T then
string, which passes over a frictionless pulley at the top of the
inclined plane of inclination q. Let the tension in the string be T.

M1 M2
N

M1 a b

q M2
M1g sinq M1g cosq FBD of M1
M1g a
T
M2g M1
q

na a M1gcosa
(i) When the mass M1 moves upwards with acceleration a. gsi M1g
From the FBD of M1 and M2, M 1

T M1g sin q = M1a ...(1) Equation of motion for M1


M2g T = M2a ...(2) T M1g sin a = M1a
Solving eqns. (1) and (2) we get, or T = M1g sin a + M1a ...(1)
M - M1 sin q FBD of M2
a= 2 g T a
M1 + M 2 M
2

FBD of mass M1
M
M2gcosb b 2 gsi
R=N y M2g nb
T x
Equation of motion for M2
M2g sinb T = M2a
M1g cos q or T = M2g sin b M2a ...(2)
M1g sin q M1g Using eqn. (1) and (2) we get,
M1g sin a + M1a = M2g sin b M2a
FBD of M2
Solving we get,
M M g
T = 2 1
M
1 + M 2 (1+sin q) ( M 2 sin b - M1 sin a ) g M 1M 2 g
T a a= and T = [sin b + sin a]
M2g M1 + M 2 M1 + M 2
134
Case 7 : A person/monkey climbing a rope (d) If angles of inclination are q1 and q2 for two inclined planes
T
t1 sin q 2
a Keeping the length constant then =
t2 sin q1

a Case 9 : Weight of a man in a lift :


(i) When lift is accelerated upward : In this case the man also
Mg r
moves in upward direction with an acceleration a .
(a) A person of mass M climbs up a rope with acceleration a.
The tension in the rope will be M(g+a).
T Mg = Ma T = M(g + a)
(b) If the person climbs down along the rope with acceleration a a
a, the tension in the rope will be M(ga).
mg
T
N
Then from Newton second law
N mg = ma or N = m(g + a)
a a
or Wapp = m(g + a) = Wo (1 + a / g ) (as W = mg)
Mg Where Wapp is apparent weight of the man in the lift, Wo is
Mg T = Ma T = M(g a) the real weight, N is the reaction of lift on the man. It is clear
(c) When the person climbs up or down with uniform speed, that N = Wapp
tension in the string will be Mg. When the lift moves upward and if we measure the weight
Case 8 : A body starting from rest moves along a smooth inclined of the man by any means (such as spring balance) then we
plane of length l, height h and having angle of inclination q. observe more weight (i.e., Wapp) than the real weight (Wo)
Wapp >Wo
(ii) When lift is accelerated downward : In this case from
l Newtons second law
h FBD of body
q
a
N=R
mg
N
q mg N = ma
mg sinq mg cosq or N = m(g a) = Wo(1 a/g)
mg
(where N=R is normal reaction applied by plane on the body
or W'app= Wo(1 a/g) {Q Wo = mg}
of mass m) If we measure the weight of man by spring balance, we
For downward motion, along the inclined plane, observe deficiency because Wapp< Wo.
mg sin q = ma a = g sin q
(iii)When lift is at rest or moving with constant velocity : From
By work-energy theorem loss in P.E. = gain in K.E. Newtons second law N mg = 0 or N = mg
1 In this case spring balance gives the true weight of the man.
mgh = mv 2 v = 2gh
2 Case 10 : Three masses M1, M2 and M3 are placed on a smooth
surface in contact with each other as shown in the figure.
Also, from the figure, h = l sin q. \ v = 2gh = 2gl sin q
A force F pushes them as shown in the figure and the three
(a) Acceleration down the plane is g sin q. masses move with acceleration a,
(b) Its velocity at the bottom of the inclined plane will be
2 gh = 2 g l sin q M3
(c) Time taken to reach the bottom will be M2
M1
1/ 2 1/ 2 F2
2l 2h 1 1 2h F1 F
t = = = = F2 F1

g sin q g sin 2 q 1/ 2 sin q g
g
sin q
2h a
135
M1
mg R l
F F1 = m1a ...(i) = 1 - cos R ;
F1 F l
M2
F2 F1 F1 F2 = m2a ...(ii) F gR l
\ Acceleration, a = = 1 - cos .
M3 m l R
F2 F2 = M3 a ...(iii)
Example 2.
Having gone through a plank of thickness h, a bullet
F
Adding eqns. (i), (ii) and (iii) we get, a = changed its velocity from u to v. Find the time of motion of
M1 + M 2 + M 3 the bullet in the plank, assuming the resistance force to be
M 3F (M 2 + M 3 )F proportional to the square of the velocity.
F2 = and F1 = Solution :
M1 + M 2 + M 3 M1 + M 2 + M 3
Given force F= - k v 2 , where k is a constant. Negative sign
Keep in Memory shows that the force is retarding one. Now, force = rate of
change of momentum = m dv/d t ;
1. When a man jumps with load on his head, the apparent
weight of the load and the man is zero. mdv 2
= - kv 2 or mdv / v = -kdt ;
2. (i) If a person sitting in a train moving with uniform dt
velocity throws a coin vertically up, then coin will fall Integrating it within the conditions of motion i.e. as time
back in his hand. changes from o to t, the velocity changes from u to v, we
(ii) If the train is uniformly accelerated, the coin will fall
have
behind him.
(iii) If the train is retarded uniformly, then the coin will fall v t v
or - m = -kt
dv 1
Example 1.
in front of him. m
u
v2 0

= - k dt ;
v u
A chain of length l is placed on a smooth spherical surface
of radius R with one of its ends fixed at the top of the mu-v
or t= ....(i)
sphere. What will be the acceleration a of each element of k uv
the chain when its upper end is released? It is assumed
dv dv ds
R . Also, F = m =m = - kv 2
that the length of chain l < dt ds dt
2
Solution : dv k ds
Let m be the mass of the chain of length l. Consider an
or = - d s. ; Q = v ; Integrating it,
v m dt
element of length dl of the chain at an angle q with vertical,
v h
dv k k h
we get u
v
=-
m
ds
0
or (log e v )uv = -
m
(s )o
dl
q dq -k kh
or log e v - log e u = ( h - 0) = -
m m
R
m
or k = log e (u / v)
h
From figure, dl = R d q ;
Putting this value in eqn. (i), we get
Mass of the element,
m m m( u - v) / uv h (u - v)
dm = dl ; or dm = . R d q t= =
l l (m / h ) log e (u / v) uv log e (u / v)
Force responsible for acceleration, dF = (dm)g sinq ; Example 3.
A wire of mass 9.8 10-3 kg per metre passes over a
m mgR
dF = R d q (g sin q) = sin q d q frictionless pulley fixed on the top of an inclined
l l frictionless plane which makes an angle of 30 with the
Net force on the chain can be obtained by integrating the horizontal. Masses M1 and M2 are tied at the two ends of
above relation between 0 to a, we have the wire. The mass M1 rests on the plane and mass M2
a a mg R
hangs freely vertically downwards. The whole system is in
mg R mg R
F= l
sin q dq =
l
(- cos q ) =
0 l
[1 - cos a] equilibrium. Now a transverse wave propagates along the
wire with a velocity of 100 ms1. Find M1 and M2.
0
136
Solution :
1 1
Resolving M1g into rectangular components, we have M1g h = 0 + g ( t / 2) 2 = gt 2 / 4 ...(ii)
sin 30 acting along the plane downwards, and M1g cos30 2 2
acting perpendicular to the plane downwards. The situation Dividing eqn. (ii) by (i), we get
has been shown in fig. h 1
= [Q cos q = h / l]
l 4 cos q
R 1 1 1
T
or cos q = ; or cos 2 q = ; or cos q =
F 4 cos q 4 2
inq or q = 60
M1 gs q
M1g cosq Example 5.
M1g M2g
q A large mass M and a small mass
m hang at the two ends of a string
Let T be the tension in the wire and R be the reaction of
that passes through a smooth
plane on the mass M1. Since the system is in equilibrium,
tube as shown in fig. The mass m
therefore,
moves around a circular path in q
T = M1g sin30 ...(i) l
a horizontal plane. The length of
and R = M1g cos30 ...(ii)
the string from mass m to the top r
T = M2g ...(iii) m
of the tube is l, and q is the angle
From eqn. (i) and (iii) we have
the string makes with the
T = M1g sin30 = M2g ...(iv)
vertical. What should be the
T frequency (n ) of rotation of mass
Velocity of transverse wave, v = , m so that mass T
m
M remains stationary? M
where m is the mass per unit length of the wire.
Solution :
2 2
\ v = T / m , or T = v m = (100)2 (9.8 10-3) = 98N Tension in the string T = Mg.
n
From eq . (iii), M2 = T/g = 98/9.8 = 10kg. Centripetal force on the body = mrw2 =mr ( 2p n )2. This is
From eqn . (iv), M1 = 2M2 = 2 10 = 20kg. provided by the component of tension acting horizontally
Example 4. i.e. T sinq ( = Mg sinq).
A block slides down a smooth inclined plane to the ground 1 Mg
when released at the top, in time t second. Another block \ mr ( 2pn)2 = Mg sinq = Mgr/l. or n =
2p ml
is dropped vertically from the same point, in the absence
Example 6.
of the inclined plane and reaches the ground in t/2 second. A string of negligible mass going over a clamped pulley of
Then find the angle of inclination of the plane with the mass m supports a block of mass M as shown in fig. The
vertical. force on the pulley by the clamp is given by
Solution :
If q is the angle which the inclined plane makes with the (a) 2 Mg
vertical direction, then the acceleration of the block sliding m
down the plane of length l will be g cosq. (b) 2 mg

A (c) [ (M + m)2 + m2 ] g
q
(d) [ (M + m)2 + M2 ] g M
l h
Solution : (c)
Force on the pulley by the clamp = resultant of
T = (M + m)g and mg acting along horizontal and vertical
C B
respectively
1 2 \ F = [(M + m)g]2 + (mg)2 = [ (M + m)2 + m2 ]g
Using the formula, s = ut + at , we have s = l, u = 0, t =
2 Example 7.
t and a = g cos q. The masses of 10 kg and 20 kg respectively are connected
1 1 by a massless spring in fig. A force of 200 newton acts on the
so l = 0 t + g cos q t 2 = (g cos q)t 2 ...(i) 20 kg mass. At the instant shown, the 10 kg mass has
2 2
acceleration 12 m/sec2. What is the acceleration of 20 kg
Taking vertical downward motion of the block, we get mass?
137

20 kg m
10 kg
200 newton
M'
Solution : q
Force on 10 kg mass = 10 12 = 120 N
M
The mass of 10 kg will pull the mass of 20 kg in the backward
direction with a force of 120 N. Solution :
\ Net force on mass 20 kg = 200 120 = 80 N
Since m does not slip on M' (relative velocity of m w.r.t. M'
force 80 N is zero)
Its acceleration a = = = 4 m / s2
mass 20 kg \ M', m will move with same acceleration as that of M.
Example 8. Since surfaces are smooth
Two masses each equal to m are lying on X-axis at (a, 0) \ frictional force is zero
and (+ a, 0) respectively as shown in fig. They are connected Net force = Mg = (M + M' + m) a
by a light string. A force F is applied at the origin and
along the Y-axis. As a result, the masses move towards each Mg
\a = ....(1)
other. What is the acceleration of each mass? Assume the M + M + m
instantaneous position of the masses as ( x, 0) and (x, 0) Now let us see m, w.r.t. M'
respectively
F N
ma

mg
q
(a, 0) (a, 0)
X X Downward acceleration of m on slope = 0
m O m
Solution : \ N ma sin q + mg cos q = 0 ....(2)
(net ^ force = 0)
F and mg sin q ma cos q = 0 ....(3)
A [Q net force along slope = 0]
T T From eqn. (3) g sin q = a cos q or a = g tan q ....(4)
q
n
From eq . (4) and (1),
B C
(x, 0) O (x, 0) M
we have tan q = M cot q = M + M' + m
M + M + m
From figure F = 2 T cos q or T = F/(2 cos q)
The force responsible for motion of masses on X-axis is T M + m
M=
sin q cot q - 1

F Example 10.
\ m a = T sin q = sin q
2 cos q Find the acceleration of block A and B. Assume pulley is
massless.
F F OB F x
= tan q = =
2 2 OA 2 (a - x 2 )
2 2kg
A
F x
so, a =
2m (a - x 2 )
2

Example 9.
Find the mass M of the hanging block in figure which will
prevent smaller block from slipping over the triangular B 5kg
block. All surfaces are frictionless and the string and the
pulley are light.
138
Solution :
If acceleration of B is a, then acceleration of A is 2a, since A
moves twice the distance moved by B
T' (T +T) = 0 (since pulley is massless)
T' N

T T Substituting value of T2 in equation (2),


T F = 4m1a + m2a = (4m1 + m2)a
B A
F 1.50
Hence a = = = 0.88 m / s 2
2g 4m1 + m 2 4(0.3) + 0.5
T'
5kg Example 13.
T' = 2T ....(1) A mass of 15 kg and another of mass 6 kg are attached to a
5gT'=5a (for 5 kg block) pulley system as shown in fig. A is a fixed pulley while B is
5g 2T = 5a ....(2) a movable one. Both are considered light and frictionless.
T= 2 (2a) = 4a ....(3) (for 2 kg block) Find the acceleration of 6 kg mass.
From equations (2) and (3),
5g (2 4a)
5g
a= A
13
10g 5g
a A = 2a = ; aB = a =
13 13
Example 11.
A block of mass M is pulled along horizontal frictionless M2
surface by a rope of mass m. Force P is applied at one end B
of rope. Find the force which the rope exerts on the block. 6kg
Solution :
The situation is shown in fig M1
T O 15kg
M P
m Solution :
Let a be the common acceleration of the system. Here
T = M a for block Tension is the same throughout the string. It is clear that
P T = m a for rope M1 will descend downwards while M2 rises up. If the
acceleration of M1 is a downwards, M2 will have an
P acceleration 2a upward.
\ P M a = m a or P = a (M + m) or a =
( M + m)
MP A
\T =
( M + m) T T T
Example 12.
In the system shown below, friction and mass of the pulley 2a M2 B
are negligible. Find the acceleration of m 2 if
m1 = 300 g, m2 = 500 g and F = 1.50 N M1 a
M2g

M1g
Now, M1g 2T = M1a
T M2g = M2.2a
Solution : or M1g 2M2g = a(M1 + 4M2)
When the pulley moves a distance d, m1 will move a distance M1 - 2M 2 15 - 12 3
a= g= g= g
2d. Hence m1 will have twice as large an acceleration as m 2 M1 + 4M 2 15 - 24 39
has.
For mass m1, T1 = m1 (2a) ...(1) g
\ a = 13
For mass m2, F T2 = m2(a) ...(2)
T2 2g
Putting T1 = in eqn. (1) gives T2 = 4m1a \ acceleration of 6 kg mass = 2a = 13
2
139

5.2
Solve following problems with the help of above text and 6. The tension in the cable of 1000 kg elevator is 1000 kg wt,
examples : the elevator
1. Tension in the cable supporting an elevator, is equal to the (a) is ascending upwards
weight of the elevator. From this, we can conclude that the (b) is descending downwards
elevator is going up or down with a (c) may be at rest or accelerating
(a) uniform velocity (b) uniform acceleration (d) may be at rest or in uniform motion
(c) variable acceleration (d) either (b) or (c) 7. Consider an elevator moving downwards with an
2. The force exerted by the floor of an elevator on the foot of acceleration a. The force exerted by a passenger of mass m
a person standing there, is more than his weight, if the on the floor of the elevator is
elevator is (a) ma (b) ma mg
(a) going down and slowing down (c) mg ma (d) mg + ma
(b) going up and speeding up 8. If an elevator is moving vertically up with an acceleration
(c) going up and slowing down a, the force exerted on the floor by a passenger of mass M
(d) either (a) or (b) is
3. A reference frame attached to earth cannot be an inertial (a) Ma(b) Mg
frame because
(c) M (g a) (d) M (g + a)
(a) earth is revolving around the sun
9. You are on a frictionless horizontal plane. How can you
(b) earth is rotating about its axis
get off if no horizontal force is exerted by pushing against
(c) Newtons laws are applicable in this frame
the surface?
(d) both (a) and (b)
(a) By jumping
4. When an elevator cabin falls down, the cabin and all the
(b) By spitting or sneezing
bodies fixed in the cabin are accelerated with respect to
(c) by rolling your body on the surface
(a) ceiling of elevator (b) floor of elevator
(d) By running on the plane
(c) man standing on earth
10. Pulling a roller is easier than pushing because
(d) man standing in the cabin
(a) when we pull a roller, the vertical component of the
5. A particle is found to be at rest when seen from frame S1 and
pulling force acts in the direction of weight
moving with a constant velocity when seen from another frame
(b) the vertical component of the pulling force acts in the
S2. Mark out the possible option.
opposite direction of weight
(a) S1 is inertial and S2 is non-inertial frame
(c) force of friction is in opposite direction
(b) both the frames are non-inertial
(d) it is possible in the case of roller only
(c) both the frames are inertial
(d) either (b) or (c).

ANSWER KEY
1. (a) 2. (b) 3. (d) 4. (c) 5. (d) 6. (d) 7. (c) 8. (d) 9. (b) 10. (b)

LAW OF CONSERVATION OF LINEAR MOMENTUM r d r r


If Fext . = 0 (P) = 0 or P = constant .....(3)
A system is said to be isolated, when no external force racts on it. dt
r
For such isolated system, the linear momentum ( P = mv ) is This is called law of conservation of momentum.
constant i.e., conserved. Now let us consider a rigid body consisting of a large number of
The linear momentum is defined as particles moving with different velocities, then total linear
r r momentum of the rigid body is equal to the summation of individual
P = mv .....(1)
r linear momentum of all particles
where v is the velocity of the body, whose mass is m. The direction n r r r r r
r p i = p1 + p 2 + p 3 + ..........p n
of P is same as the direction of the velocity of the body. It is a i.e.,
i =1
vector quantity. From Newtons second law,
r n r r r r r
r d r d r or Ptotal = pi = p1 + p2 + p3 + .......... + pn
Fext . = (mv) = P .....(2) i =1
dt dt r r r
i.e., time rate of change in momentum of the body is equal to total where p1, p 2 ...............p n are individual linear momentum of first,
external force applied on the body. second and nth particle respectively.
140
If this rigid body is isolated i.e., no external force is applied on it, Force vary with time and impulse is area under force versus
r time curve
then Ptotal = constant (from Newtons second law).
Further we know that internal forces (such as intermolecular forces Fext.
etc.) also act inside the body, but these can only change individual
linear momentum of the particles (i.e., p1, p2.........), but their total Area=Fext.Dt
r
momentum Ptotal remains constant. Fext.
Gun Firing a Bullet Fav
.
If a gun of mass M fires a bullet of mass m with velocity v. Then t
from law of conservation of momentum, as initially bullet & gun ti (b) tf
are at rest position i.e., initial momentum is zero, so final momentum r
(gun + bullet) must also be zero. Force constant with time i.e., Fext. constant with time (shown
r
Since on firing, the bullet moves with velocity v b in forward by horizontal line) and it would give same impulse to particle
direction, then from Newtons third law, the gun moves in backward in time Dt = tf ti as time varying force described.
r It is a vector quantity having a magnitude equal to the area under
direction v g . So, the force-time curve as shown in fig. (a). In this figure, it is assumed
Initial momentum = final momentum that force varies with time and is non-zero in time interval Dt = tf
r r
r uuur ti. Fig.(b) shows the time averaged force Fext. i.e., it is constant
0 = mvb + MVg uuur - mvb
Momentum Momentum \ V =
g
in time interval Dt, then equation (iii) can be written as
of bullet of gun M r r
t r
(ve sign shows that the vel. of gun will have the opposite I = Fext. f dt = F (t - t ) I = F ext. Dt ...(iv)
t
i ext. f i
direction to that of bullet) The direction of impulsive vector I is same as the direction of
IMPULSE change in momentum. Impulse I has same dimensions as that of
According to Newtons second law the rate of change of momentum i.e, [MLT1]
momentum of a particle is equal to the total external force applied Rocket propulsion (A case of system of variable mass ) : It is
on it (particle) i.e., based on principle of conservation of linear momentum.
r
dP r In rocket, the fuel burns and produces gases at high temperature.
= Fext ...(i) These gases are ejected out of the rocket from nozzle at the
dt
backside of rocket and the ejecting gas exerts a forward force on
r r r r r tf r the rocket which accelerates it.
or dP = Fext .dt or DP = Pf - Pi = Fext .dt ...(ii)
t i dM
r Let the gas ejects at a rate r = - and at constant velocity u
Where Pi is momentum of the particle at initial time ti and when dt
r r w.r.t. rocket then from the conservation of linear momentum
we apply some external force Fext its final momentum is Pf at
dv ru ru
r = = where M = M0 - rt and M0 is mass of rocket
time tf . The quantity Fext dt on R.H.S in equation (ii) is called dt M M 0 - rt
the impulse.
We can write equation (ii) as M0
with fuel and solving this equation, we get v = u log e

t r r M 0 - rt
I = f Fext .dt = DP ...(iii) where v = velocity of rocket w.r.t. ground.
ti
r Example 14.
So, the impulse of the force Fext is equal to the change in Two skaters A and B approach each other at right angles.
momentum of the particle. It is known as impulse momentum Skater A has a mass 30 kg and velocity 1 m/s and skater
theorem. B has a mass 20 kg and velocity 2 m/s. They meet and
cling together. Find the final velocity of the couple.
Fext. Solution :
Applying principle of conservation of linear momentum,

p = p12 + p 22 ; ( m1 + m 2 ) v = ( m1 v1 ) 2 + ( m 2 v 2 ) 2
impulse

( 30 + 20 ) v = ( 30 1)2 + ( 20 2 )2 = 50
Area=

50
t v= =1 m / s
ti (a) tf 50
141
Example 15. According to law of conservation of liner momentum, Initial
A bullet of mass M is fired with a velocity of 50 m/sec at momentum = zero \ final momentum = 0
an angle q with the horizontal. At the highest point of its
trajectory, it collides head on with a bob of mass 3M \ p1 + p 2 + p 3 = 0
suspended by a massless string of length 10/3 m and gets
embedded in the bob. After the collision, the string moves | p 3 | = = ( 21) 2 + ( 21) 2 = 21 2
to an angle of 120. What is the angle q ?
Solution : \ 3 V = 21 2 or V = 7 2 = 9.8 m / sec .

120 And it is at an angle of 135 with the direction of p1 .


Example 17.
A hammer of mass M strikes a nail of mass m with velocity
l = 10/3m
of u m/s and drives it s meters in to fixed block of wood.
Find the average resistance of wood to the penetration of
nail.
u Solution :
3M
Applying the law of conservation of momentum,

M
q m u = (M + m) v0 v 0 = u
m+M
There acceleration a can be obtained using the formula
Vel. of bullet at highest point of path = 50 cos q (v2 = u2 + 2as).
From law of conservation of linear momentum, Here we have 0 v02 = 2as or a = v02 /2s
V cos q 50 cos q
MV cos q = (3M + M) V or V= =
M u
2 2
4 4 \ a =
1 m + M 2s
Again, (M + 3M )V2 = (M + 3M )gl(1 - cos 120 )
2
M2 u2
V'2
1 Resistance = (M + m) a =
= gl 1 + or V ' = 3gl ; m+M 2s

2 2
Example 18.
50 cos q 10 A ball of mass 0.5 kg is thrown towards a wall so that it
\ = 3 10 = 10
4 3 strikes the wall normally with a speed of 10 ms1. If the ball
bounces at right angles away from the wall with a speed of
4 4
cos q = \ q = cos -1 8ms1, what impulse does the wall exert on the ball ?
5 5
Solution :
Example 16.
A body of mass 5 kg which is at rest explodes into three Approaching wall
fragments with masses in the ratio 1 : 1 : 3. The fragments u = 10 ms1
with equal masses fly in mutually perpendicular directions 10
with speeds of 21 m/sec. What will be the velocity of the f
heaviest fragment? 8 Leaving wall
Solution :
Momentum of first body v = +8 ms1
p1 = 1 21 = 21 kg m /sec.
Momentum of second body, p2 = 1 21 = 21 kg m/sec. Taking the direction of the impulse J as positive and using
Momentum of third body p3 = 3V kg m/sec J = mv mu
1 1
we have J= 8- (-10) = 9 N-s
45 2 2
p2
Therefore the wall exerts an impulse of 9 N-s on the ball.
p3

Example 19.
45
Two particles, each of mass m, collide head on when their
speeds are 2u and u. If they stick together on impact, find
135 p1 their combined speed in terms of u.
142
Solution : 1
m m (m) (2u) mu = 2m V V = u
2
Before impact u
2u The combined mass will travel at speed u/2.
(Note that the momentum of the second particle before impact
2m is negative because its sense is opposite to that specified
After impact v as positive.)
Using conservation of linear momentum (in the direction of
the velocity 2u) we have

5.3
Solve following problems with the help of above text and 3. Rocket works on the principle of
examples : (a) conservation of mass
1. A machine gun of mass M fires n bullets per second. The (b) conservation of linear momentum
mass and speed of each bullet is m and v respectively. The (c) conservation of energy
force exerted on the machine gun is (d) conservation of angular momentum
(a) zero (b) mvn 4. A bullet of mass 10 gm is fired from a gun of mass 1 kg. If
(c) Mvn (d) Mvn/m the recoil velocity is 5 ms1, the velocity of muzzle is
2. A body whose momentum is constant must have constant (a) 0.05 ms1 (b) 5 ms1
(c) 50 ms 1 (d) 500 ms1
(a) velocity (b) force
(c) acceleration (d) All of the above

ANSWER KEY

1. (b) 2. (a) 3. (b) 4. (d)


FRICTION r
(c) A graph Fext . versus | f | shown in figure. It is clear that
When a body is in motion on a rough surface, or when an object
moves through water (i.e., viscous medium), then velocity of the fs, ,max > fk
body decreases constantly even if no external force is applied on
the body. This is due to friction. |f|
So an opposing force which comes into existence, when two
surfaces are in contact with each other and try to move relative (fs)max
Body is Body starts with
to one another, is called friction. =msN at rest acceleration
Frictional force acts along the common surface between the two
bodies in such a direction so as to oppose the relative movement
of the two bodies. fk=mk N
(a) The force of static friction fs between book and rough
surface is opposite to the applied external force Fext. The O kinetic
r static
force of static friction fs = Fext . region region
(c)
R=N
Fig.(a) shows a book on a horizontal rough surface. Now if
fs Book Fext. r
we apply external force Fext. , on the book, then the book
r
(a) will remain stationary if Fext. is not too large. If we increase
r W r
(b) When Fext . exceeds the certain maximum value of static Fext. then frictional force f also increase up to (fs )max
friction, the book starts accelerating and during motion (called maximum force of static friction or limiting friction)
Kinetic frictional force is present. r
and (fs )max = msN. At any instant when Fext. is slightly
R=N Body just starts moving
greater than (fs )max then the book moves and accelerates to
Book the right.
fk Fext.
Fig.(b) when the book is in motion, the retarding frictional
(b) force become less than, (fs )max
W
143
Fig.(c) (fs )max is equal to mkN. When the book is in motion, Laws of limiting friction :
we call the retarding frictional force as the force of kinetic (i) The force of friction is independent of area of surfaces
friction fk. in contact and relative velocity between them (if it is
not too high).
Since fk< (fs )max , so it is clear that, we require more force to (ii) The force of friction depends on the nature of material
start motion than to maintain it against friction. of surfaces in contact (i.e., force of adhesion).
By experiment one can find that (fs )max and f k are m depends upon n ature of the surface. It is
independent of the normal reaction.
proportional to normal force N acting on the book (by rough (iii) The force of friction is directly proportional to normal
surface) and depends on the roughness of the two surfaces reaction i.e., F N or F = mn.
in contact. While solving a problem having friction involved, follow
Note : the given methodology
(i) The force of static friction between any two surfaces
r If Fapp < fl
in contact is opposite to Fext. and given by f s ms N Body does not move and
and (fs )max = ms N (when the body just moves in the Fapp = frictional force
right direction).
where N = W = weight of book and ms is called Check If Fapp = fl
coefficient of static friction, fs is called force of static (a) Fapp Body is on the verge of movement
(b) Limiting if the body is initially at rest
friction and (fs )max is called limiting friction or friction (fl) Body moves with constant velocity
maximum value of static friction.
(ii) The force of kinetic friction is opposite to the direction
of motion and is given by fk = mkN
where mk is coefficient of kinetic friction.
(iii) The value of mk and ms depends on the nature of
surfaces and mk is always less then ms. Rolling Friction :
Friction on an inclined plane : Now we consider a book on an The name rolling friction is a misnomer. Rolling friction has nothing
inclined plane & it just moves or slips, then by definition to do with rolling. Rolling friction occurs during rolling as well as
sliding operation.
(f s) m
a x
R=N
ok
Bo
in q q mg cos q
s mg=W
mg q Cause of rolling friction : When a body is kept on a surface of
another body it causes a depression (an exaggerated view shown
( f s )max = m s R in the figure). When the body moves, it has to overcome the
Now from figure, f s,max = mg sin q and R = mg cosq depression. This is the cause of rolling friction.

ms= tanq or q = tan1(ms) Rolling friction will be zero only when both the bodies
where angle q is called the angle of friction or angle of repose incontact are rigid. Rolling friction is very small as compared to
sliding friction. Work done by rolling friction is zero
Some facts about friction :
(1) The force of kinetic friction is less than the force of static CONSERVATIVE AND NON-CONSERVATIVE FORCES
friction and the force of rolling friction is less than force of If work done on a particle is zero in complete round trip, the
kinetic friction i.e., force is said to be conservative. The gravitational force,
fr < fk < fs or mrolling < mkinetic < mstatic electrostatics force, elastic force etc., are conservative forces.
hence it is easy to roll the drum in comparison to sliding it. On the other hand if the work done on a body is not zero during
(2) Frictional force does not oppose the motion in all cases, a complete round trip, the force is said to be non-conservative.
infact in some cases the body moves due to it. The frictional force, viscous force etc. are non-conservative
forces.
B
Final
position
A Fext A f
In the figure, book B moves to the right due to friction B
between A and B. If book A is totally smooth (i.e., frictionless) C
i
then book B does not move to the right. This is because of Initial
no force applies on the book B in the right direction. position
144
Figure shows three processes A, B and C by which we can reach Example 21.
from an initial position to final position. If force is conservative, An object of weight W is resting on an inclined plane at an
then work done is same in all the three processes i.e., independent angle q to the horizontal. The coefficient of static friction
of the path followed between initial and final position. is m . Find the horizontal force needed to just push the object
up the plane.
If force is non conservative then work done from i to f is different
Solution :
in all three paths A,B and C.
The situation is shown in fig.
Hence it is clear that work done in conservative force depends
only on initial & final position irrespective of the path followed osq
R Fc
between initial & final position. In case of non-conservative q F
forces the work done depends on the path followed between
sinq
initial and final position. W q F sinq
f=m R
q W cosq
We can say also that there is no change in kinetic energy of the
body in complete round trip in case of conservative force. While W
in case of non conservative forces, when a body return to its
Let F be the horizontal force needed to just push the object
initial position after completing the round trip, the kinetic energy up the plane. From figure R = W cos q + F sin q
of the body may be more or less than the kinetic energy with Now f = mR = m [W cos q + F sin q] ...(1)
which it starts.
Further, F cos q = W sin q + f ...(2)
Example 20. F cos q = W sin q + m [W cos q + F sin q]
Pushing force making an angle q to the horizontal is F cos q m F sin q = W sin q + m W cos q
applied on a block of weight W placed on a horizontal W (sin q + m cos q)
table. If the angle of friction is f , then determine the \ F=
(cos q - m sin q)
magnitude of force required to move the body.
Example 22.
Solution :
A block A of mass m1 rests on a block B of mass m2. B rests
The various forces acting on the block are shown in fig. on fixed surface. The coefficient of friction between any
two surfaces is m . A and B are connected by a massless
N string passing around a frictionless pulley fixed to the wall
as shown in fig. With what force should A be dragged so as
f cosq to keep both A and B moving with uniform speed?

f q
F
mg F sinq
T
F A
Here, f2 f1

f B
m = tan f = ; or f = N tanf ...(i) T
N f3

The condition for the block just to move is Solution :


Fcosq = f = N tanf ...(ii) The situation is shown in fig.
Let F be the horizontal force applied on A.
and F sinq + W = N ...(iii) For block A, F = T + f1 = T + m m1g .....(1)
From (ii) and (iii), (Q Block A moves towards left, frictional force f1 acts
towards right)
F cosq = (W + F sinq ) tan f = W tanf + F sin q tanf ;
For block B, fB = f2 + f3
or F cos q F sinq sinf/cosf = W sinf/cosf (Q Block B moves towards right, frictional forces f2 and f3
or F (cosq cosf sinq sinf) = W sinf ; acts towards left).
T = m m1g + m (m1 + m 2 ) g = m g (2 m1 + m 2 ) ...(2)
or F cos (q + f) = W sinf or F = W sinf / cos (q + f)
From eqns. (1) and (2), we get
F = m g (2 m1 + m 2 ) + m m1 g or F = m g (3 m1 + m 2 )
145
Example 23. Example 24.
Figure shows a small block of mass m kept at the left hand Find the acceleration of the block of mass M in the situation
of a larger block of mass M and length l. The system can of figure. The coefficient of friction between the two blocks
slide on a horizontal road. The system is started towards is m 1 and between the bigger block and the ground is m 2.
right with an initial velocity v. The friction coefficient
between road and bigger block is m and between the block
is m /2. Find the time elapsed before the smaller block
separates from the bigger block. m
M
m
M Solution :
We make free body diagram of mass M and m separately,
Solution : Let acceleration of M be a, then acceleration of m w.r.t. M
Make free body diagram of m will be 2a since m moves twice the distance moved by m
Take right as the positive direction. Let a 1/g be the N'
T
acceleration of m w.r.t. ground.
M T
N N T+T
N N' f1 f2+N
N T
m f1 f2 Mg Mg+f1+T
M
Now see m w.r.t. M
f1 f2 Mg T
mg f1

-f1 m
Q f1 = 2 mg
a1 / g = ma N
m
\ N = ma ...(1)
-mg mg
f1 = m1N = m1ma ...(2)
a1/ g = ...(1)
2 mg f1 T = m(2a) mg = m1ma + T + 2ma
N' = N + Mg and N' = (m + M)g ...(2) mg T = (2 + m1)ma ...(3)
f2 = mN' = m(m +M)g ...(3) or T = mg (2 + m1)ma ...(4)
-(f 2 - f1 ) for M, N' = Mg + f1 + T = Mg + mma + T ...(5)
a2/g =
M and 2T (f2 + N) = Ma
[Q a2/g is acceleration of M w.r.t. ground] 2T m2(N') N = Ma ...(6)
-{m(m + M)g - m / 2 mg} m 2T m2(Mg + m1 ma + T) ma = Ma
= = -mg 1 + [Using eqns.(5) and (6)]
M 2M
a1/2 = acceleration of m w.r.t. to M = a1/g a2/g (2 m2)T = m2Mg + m1m2 ma + (M + m)a ...(7)
Solving equation (4) and (7), we get
mg m 1 m [m + M]
=- + mg 1 + = mg - + 1 + = mg
2 2M 2 2M 2M a=
[2m - m 2 (M + m)]g
M + m[5 + 2(m1 - m 2 )]
1 4Ml
Now l = a1/ 2 t 2 t =
2 (M + m)mg

5.4
Solve following problems with the help of above text and 2. Which of the following is a self adjusting force?
examples : (a) Static friction (b) Limiting friction
1. Which of the following statements about friction is true? (c) Dynamic friction (d) Sliding friction
(a) Friction can be reduced to zero 3. The force required to just move a body up the inclined plane
(b) Frictional force cannot accelerate a body is double the force required to just prevent the body from
(c) Frictional force is proportional to the area of contact sliding down the plane. The coefficient of friction is m. The
between the two surfaces inclination q of the plane is
(d) Kinetic friction is always greater than rolling friction (a) tan1 m (b) tan1 (m/2)
1
(c) tan 2m (d) tan1 3m
146

4. If ms, mk and mr are coefficients of static friction, sliding (a) sin q (b) cos q
friction and rolling friction, then (c) tan q (d) independent of q
(a) ms < mk < mf (b) mk < mr < ms 7. Which of the following statements is correct, when a
(c) mr < mk < ms (d) mr = mk = ms person walks on a rough surface?
5. A 30 kg block rests on a rough horizontal surface. A force of (a) The frictional force exerted by the surface keeps him
200 N is applied on the body. The block acquires a speed of moving
4 m/sec, starting from rest, in 2 seconds. What is the value of (b) The force which the man exerts on the floor keeps
coefficient of friction? him moving
(c) The reaction of the force which the man exerts on
(a) 10 / 3 (b) 3 / 10 floor keeps him moving
(c) 0.47 (d) 0.185 (d) None of these
6. A block is at rest on an inclined plane making an angle a 8. It is difficult to move a cycle with brakes on because
with the horizontal. As the angle a of the inclination is (a) rolling friction opposes motion on road
increased, the block just starts slipping when the angle of (b) sliding friction opposes motion on road
inclination becomes q. Then the coefficient of static (c) rolling friction is more than sliding friction
friction between the block and the surface of the inclined (d) sliding friction is more than rolling friction
plane is
ANSWER KEY
1. (d) 2. (a) 3. (d) 4. (c) 5. (c) 6. (c) 7. (c) 8. (d)

CASES OF CIRCULAR MOTIONS


Motion in a Vertical Circle : or, v 2A - v 2B = 4gR ...(4)

Let us consider a particle of mass m attached to a string of length (change in kinetic energy of particle)
R let the particle be rotated about its centre O. = (change in potential energy of particle)
At t = 0 the particle start with velocity u from the point A (lowest or
point of vertical circle) and at time t its position is P. Then the (loss in kinetic energy of the particle) = (gain in potential energy)
tension at point P is given by
In conservative force system (such as gravity force) the mechanical
B energy (i.e., kinetic energy + potential energy) must be constant.
Total energy will be constant
Now from eqns.(2) and (3), we get
B
O vP vB B
q T
R P mg R
D
q TB
A u mg cos q D TA
mg sin q mg R

A vA= vC
mv 2P mv2P
TP - mg cos q = or TP = mg cos q + ...(1) A mg
R R m m
So tension at point A (lowest point of vertical circle) is TA - TB = 2mg + (VA2 - VB2 ) = 2mg + (4gR )
R R
mv 2A
TA - mg = (Q q = 0) ...(2) TA - TB = 6mg ...(5)
R
and tension at point B (highest point of vertical circle) is or TA = TB + 6mg ...(6)
So it is clear from eqn. (6) that tension in string at lowest point
mv 2B of vertical circle is greater then the tension at highest point of
TB + mg = (Q q =180) ...(3)
R vertical circle by 6mg.
mv 2 Condition to complete a vertical circle :
Where is centripetal force required for the particle to move If we reduce the velocity vA in equation (2), then TA will be reduce
r
and at some critical velocity vc, TB will be zero, then put TB = 0
in a vertical circle.
and vB = vC in equation (3) and we obtain
Now from law of conservation of energy
vC = vB = gR ...(7)
1 1
mv 2A - mv 2B = 2 mgR
2 2
147
In this condition the necessary centripetal force at point B is Solution :
provided by the weight of the particle [see again equation (3)]
P
then from equation (4), we get
VP
v 2A - gR = 4 gR v A = 5 gR ...(8) L
then the tension at the point A will be
O
m(5gR ) T
TA = mg + = 6mg ...(9) q
R mg cos q
Hence if we rotate a particle in a vertical circle and tension in
VO q
string at highest point is zero, then the tension at lowest point of Q
vertical circle is 6 times of the weight of the particle. mg
The tension T in the string is given by
Some Facts of Vertical Motion :
vQ2
(i) The body will complete the vertical circle if its velocity at vP 2
Tmax = m g + and T
min = m - g +
L L
lowest point is equal to or greater then 5gR
(ii) The body will oscillate about the lowest point if its velocity According to the given problem
at lowest point is less then 2 gR . This will happen when g + ( v Q 2 / L)
= 4 or vQ 2 vP2
the velocity at the halfway mark, i.e. 2 g+ = -4 g + 4
- g + ( v P / L) L L
1
v D = 0 Q mv 2A = mgR
2 vP 2 + 4 g L v 2
or g+ = -4 g + 4 P
(iii) The string become slack and fails to describe the circle L L
when its velocity at lowest point lies between L = (10/3) m and g = 10 m/s2 (given)
2gR to 5gR Solving we get vP = 10 m/s.

Example 25. Negotiating a Curve :


A mass m is revolving in a vertical circle at the end of a Case of cyclist
string of length 20 cm. By how much does the tension of To safely negotiate a curve of radius r, a cyclist should bend at
the string at the lowest point exceed the tension at the an angle q with the vertical.
topmost point? N Ncosq
Solution :
The tension T 1 at the topmost point is given by,
q
m v12
T1 = -mg
20
Nsinq
Centrifugal force acting outward while weight acting
downward
m v 22
The tension T2 at the lowest point, T2 = + mg v2
20
Which is given by tan q = . Angle q is also called as angle of
Centrifugal force and weight (both) acting downward rg
m v 2 2 - m v12 banking.
T2 - T1 = + 2mg ; v12 = v 2 2 - 2 g h or
20
mv 2
2 2
v 2 - v1 = 2 g (40 ) = 80 g N sin q = and N cos q = mg
r
80 m g Case of car on a levelled road
\ T2 - T1 = + 2mg = 6 mg
20 A vehicle can safely negotiate a curve of radius r on a rough
Example 26. level road when coefficient of sliding friction is related to the
A stone of mass 1 kg tied to a light inextensible string of v2
length L = (10/3) m is whirling in a circular path of radius velocity as m s .
rg
L in a vertical plane. If the ratio of the maximum to the
minimum tension in the string is 4 and g = 10 m/s2, then find Now consider a case when a vehicle is moving in a circle, the
the speed of the stone at the highest point of the circle.
148

mv 2 rg (m + tan q)
centrifugal force is whereas m is mass of vehicle, r = radius v2 = ; where m is the coefficient of friction of the
r 1 - m tan q
of circle and v is its velocity. rough surface on which the vehicle is moving, and q is the angle
of inclined road with the horizontal.
mv 2 Suppose a vehicle is moving in a circle of radius r on a rough
r
inclined road whose coefficient of friction is and angle of
fs M
banking is q.
The frictional force is static since wheels are in rolling motion
N
because point of contact with the surface is at rest N

mv 2 mv 2 mv 2
\ fs = f s f max = m s mg r r
r
fs mg mg
mv 2 v2 fs
m s mg or m s q
r rg
Case of banking of road (frictionless) Let velocity of object (vehicle) be V.
A vehicle can safely negotiate a curve of radius r on a smooth mv2
(frictionless) road, when the angle q of banking of the road is If we apply pseudo force on body, centrifugal force is
r
v2 when v is max. and friction force will be acting down the slope.
given by tan q = .
rg mv2
Balancing the force horizontally, = f s cos q + N sin q ...(1)
r
Balancing the force vertically,
N N cos q = f s sin q + mg ...(2)
q
Vertical

when v = maximum, f = fmax = fs = mN ...(3)


From eqn. (2),
q mg N cos q = mN sin q + mg N (cos q - m sin q) = mg
Horizontal mg
or N =
When the banked surface is smooth, the force acting will be gravity cos q - m sin q
and normal force only.
mv 2 mmg cos q + mg sin q
From eqns.(1) and (3), =
N r cos q - m sin q

mv2 mv 2 mg (m + tan q) (m + tan q)


2
r = vmax = rg
r 1 - m tan q 1 - m tan q
mg Now in the case of minimum velocity with which body could
q
move in a circular motion, the direction of friction will be opposite
to that one in maximum velocity case.
Balancing forces
N cos q = mg ...(1) fs
N
mv 2
N sin q = ...(2) mv 2
r
r
v2
= tan q ...(3) mg
rg q
Case of banking of road (with friction)
The maximum velocity with which a vehicle can safely negotiate 2 m - tan q
a curve of radius r on a rough inclined road is given by and vmin = rg
1 + m tan q
149
Keep in Memory CONICAL PENDULUM
Consider an inextensible string of length l which is fixed at
1. Whenever a particle is moving on the circular path then
one end, A. At the other end is attached a particle P of mass
there must be some external force which will provide the
m describing a circle with constant angular velocity w in a
necessary centripetal acceleration to the particle.
horizontal plane.
For examples :
(i) Motion of satellite around a planet : Here the centripetal
A
force is provided by the gravitational force.

V h Tsin
P O
2 Satellite r 2
GMm mv P O
i.e. 2
= (m) r
r r (M) Planet

mg Vertical section Horizontal Plane


(ii) Motion of electron around the nucleus : Here the
required centripetal force is provided by the As P rotates, the string AP traces out the surface of a cone.
Coulombian force Consequently the system is known as a conical pendulum.
1 (ze)(e) mv2 Vertically, T cos q = mg ... (1)
i.e. =
4pe o r 2 r
Horizontally, Tsin q = mrw 2 ... (2)
In triangle AOP, r = l sin q ... (3)
and h = l cos q ... (4)
Several interesting facts can be deduced from these
Nucleus equations :
Electron
r (a) It is impossible for the string to be horizontal.
(Ze) (e)
mg
This is seen from eqn. (1) in which cos q = cannot be
T
(iii) Motion of a body in horizontal and vertical circle: zero. Hence q cannot be 90.
Here the centripetal force is provided by the tension. (b) The tension is always greater than mg.
Horizontal circle This also follows from eqn. (1) as cos q < 1 (q is acute but
not zero). Hence, T > mg
V (c) The tension can be calculated without knowing the
mv 2 inclination of the string since, from eqn. (2) and (3)
T= (m) T sin q = ml sin q w 2 T = mlw 2
r
T
(d) The vertical depth h of P below A is independent of the
length of the string since from eqn. (1) and (4)

Vertical circle h lmg


T = mg T = but T = mlw 2
l h
mv A 2
At point A, TA = ; mlg g
r VB Therefore mlw 2 = h= 2
B h w
mg
V which is independent of l.
TB Example 27.
mv B 2 A A table with smooth horizontal surface is fixed in a cabin
At point B, TB + mg =
r T that rotates with a uniform angular velocity w in a circular
mg path of radius R. A smooth groove AB of length L (<<R) is
TC
V made on the surface of the table. The groove makes an
mv C 2 C mg C angle q with the radius OA of the circle in which a particle
And at point C, TC - mg = is kept at the point A in the groove and is released to move
r
along AB. Find the time taken by the particle to reach the
point B.
150
Let v be the velocity of the bob at the lowest position. In
B this position, The P.E. of bob is converted into K.E. Hence,
O A q 1
R mgl = m v 2 or v 2 = 2 g l ...(1)
2
If T be the tension in the string, then
Solution :
Now let us take the cabin as reference frame. Since it is m v2
accelerated we have to use pseudo force to apply Newtons T - mg =
...(2)
l
second law.
Here R >> L mw2 (R + x cos q) mw2 R ns
From eq . (1) and (2).
T m g = 2 m g or T = 3 m g

Example 30.
2 2 A light rod of length l is free to rotate in vertical plane
mw (R+x cos q) mw R about one end. A particle of mass m is attached to the other
x end.
When the rod is hanging at rest vertically downward, an
q impulse is applied to the particle so that it travels in
complete vertical circles. Find the range of possible values
Since groove is smooth (friction is zero)
of the impulse and the tangential acceleration when the
\ Component of mw2R in the direction of groove is the net
force (rest is balanced by normal force) rod is inclined at 60 to the downward vertical.
Let a is acceleration in the direction of groove
\ a' = w2R cos q
P.E. zero
2L v/
1
\ L = a t 2 t = 2
2 w R cos q
Example 28. dv
J dt
A particle of mass m is moving in a circular path of constant
radius r such that its centripetal acceleration ac is varying Solution : mg
with time t as ac = k2rt2, where k is a constant. Determine First, using impulse = change in momentum we have
the power delivered to the particle by the forces acting on J = mu ......... (1)
it. Using conservation of mechanical energy gives
Solution : 1 1
mu 2 - mgl = mv 2 - mgl cos q ......... (2)
Here tangential acceleration also exists which requires power. 2 2
Given that centripetal acceleration Applying Newton's law tangentially gives
ac = k2rt2 also, ac = v2/r ;
dv
\ v2/r = k2rt2 or v2 = k2r2t2 or v = k r t ; -mg sin q = m ......... (3)
dt
Tangential acceleration, a = dv = k r If the particle is to describe complete circles,
dt v > 0 ; q = 180
Now, force F = ma = m k r ; When q = 180 , eqn. (2) gives
So, power, P = F v = m k r k r t = m k2 r2 t.
v 2 = u 2 - 2gl + 2gl cos180 v 2 = u 2 - 4gl
Example 29.
The string of a pendulum is horizontal. The mass of the bob But v > 0 therefore, u 2 > 4gl u > 2 gl
is m. Now the string is released. What is the tension in the (u cannot be negative)
string in the lowest position?
Solution : Hence, from eqn. (1) J > 2m gl
O
3 dv
When q = 60, equation (3) becomes mg =m
2 dt
So the tangential acceleration is g 3 / 2
T

mg
151

5.5
Solve following problems with the help of above text and 7. A car takes a circular turn with a uniform speed u. If the
examples : reaction at inner and outer wheels be denoted by R1 and
1. On a railway curve the outside rail is laid higher than the R2, then
inside one so that resultant force exerted on the wheels of (a) R1 = R2 (b) R1 < R2
the rail car by the tops of the rails will (c) R1 > R2 (d) None of these
(a) have a horizontal inward component 8. A piece of stone is thrown from the top of a tower with a
(b) be vertical horizontal speed of 10 3 m/s. It is found that at a point
(c) equilibriate the centripetal force P along the path, the velocity vector of the stone makes
(d) be decreased an angle of 30 with the horizontal. The point P is reached
2. A car moving on a horizontal road may be thrown out of in time t which is given by (g = 10 m/s2)
the road in taking a turn
(a) 1 sec (b) 3 sec
(a) by the gravitational force
(b) due to the lack of proper centripetal force (c) 2 sec (d) 2 3 sec
(c) due to the rolling frictional force between the tyre 9. A block of mass m at the end of a string is whirled round in
and road a vertical circle of radius R. The critical speed of the block
(d) due to the reaction of the ground at the top of its swing below which the string would slacken
3. A cyclist taking turn bends inwards while a car passenger before the block reaches the top is
taking the same turn is thrown outwards. The reason is (a) R g (b) (R g)2
(a) car is heavier than cycle (c) R/g (d) Rg
(b) car has four wheels while cycle has only two
10. A sphere is suspended by a thread of length l. What
(c) difference in the speed of the two minimum horizontal velocity has to be imparted to the
(d) cyclist has to counteract the centrifugal force while sphere for it to reach the height of the suspension?
in the case of car only the passenger is thrown by (a) g l (b) 2 g l
this force
4. A car sometimes overturns while taking a turn. When it (c) gl (d) 2g l
overturns, it is 11. A particle rests on the top of a hemisphere of radius R.
(a) the inner wheel which leaves the ground first Find the smallest horizontal velocity that must be imparted
(b) the outer wheel which leaves the ground first to the particle if it is to leave the hemisphere without sliding
(c) both the wheel leave the ground simultaneously down is
(d) either wheel will leave the ground first (a) gR (b) 2g R
5. A tachometer is a device to measure
(a) gravitational pull (b) speed of rotation (c) 3g R (d) 5g R
(c) surface tension (d) tension in a spring 12. A body of mass m is rotated in a vertical circle of radius r.
6. A particle moves in a circle with a uniform speed. When it The minimum velocity of the body at the topmost position
goes from a point A to a diametrically opposite point B, for the string to remain just stretched is
r r
the momentum of the particle changes by p A - p B = 2 (a) 2g r (b) gr

kg m/s ( j) and the centripetal force acting on it changes (c) 3g r (d) 5g r


r r
by FA - FB = 8N (i ) where i , j are unit vectors along X 13. A small body of mass m slides down from the top of a
hemisphere of radius r. The surface of block and
and Y axes respectively. The angular velocity of the particle hemisphere are frictionless.
is
(a) dependent on its mass
(b) 4 rad/sec
r
2 h
(c) rad/sec (d) 16p rad/sec
p
152

The height at which the body lose contact with the surface 19. A can filled with water is revolved in a vertical circle of
of the sphere is radius 4 metre and the water just does not fall down. The
(a) (3/2) r (b) (2/3) r time period of revolution will be
(c) (1/2) g r2 (d) v2 /2g (a) 1 sec (b) 10 sec
14. A particle of mass m is describing a circular path of radius (c) 8 sec (d) 4 sec
r with uniform speed. If L is the angular momentum of the 20. A motor cyclist moving with a velocity of 72 km per hour
particle about the axis of the circle, the kinetic energy of on a flat road takes a turn on the road at a point where the
the particle is given by radius of curvature of the road is 20 metres. the acceleration
(a) L2/m r2 (b) L2/2 m r2 due to gravity is 10m/sec2. In order to avoid skidding, he
2
(c) 2 L /m r 2 (d) m r2 L must not bend with respect to the vertical plane by an
15. A car is travelling with linear velocity v on a circular road angle greater than
of radius r. If it is increasing its speed at the rate of a (a) q = tan1 6 (b) q = tan1 2
metre/sec2, then the resultant acceleration will be 1
(c) q = tan 25.92 (d) q = tan1 4
21. The kinetic energy K of a particle moving along a circle of
v 2
2
v 4
2 radius R depends on the distance covered s as K = a s2.
(a) 2 -a (b) 2 +a
r r The force acting on the particle is
(a) 2 a s2/R (b) 2 a s[1 + (s2/R2)]1/2
v 4 v 2 (c) 2 a s (d) 2 a R2/s
2 2
(c) 2 -a (d) 2 +a 22. A train is moving with a speed of 36 km/hour on a curved
r r path of radius 200 m. If the distance between the rails is 1.5
16. An automobile of mass m is crossing over a convex m, the height of the outer rail over the inner rail is
upwards over bridge with a speed v. If the radius of the (a) 1 m (b) 0.5 m
bridge is r, the thrust on the bridge at the highest point will (c) 0.75 m (d) 0.075 m
be 23. A mass m is revolving in a vertical circle at the end of a
string of length 20 cm. By how much does the tension of
m v2 m v2 the string at the lowest point exceed the tension at the
(a) mg + (b) m g -
r r topmost point?
(a) 2 m g (b) 4 m g
m2 v 2 g v2 g
(c) (d) (c) 6 m g (d) 8 m g
r r 24. The string of a pendulum of length l is displaced through
17. The coefficient of friction between the rubber tyres and 90 from the vertical and released. Then the minimum
the road way is 0.25. The maximum speed with which a car strength of the string in order to withstand the tension as
can be driven r ound a curve of radius the pendulum passes through the mean position is
20 m without skidding is (g = 9.8 m/s2) (a) 3 m g (b) 4 m g
(a) 5 m/s (b) 7 m/s (c) 5 m g (d) 6 m g
(c) 10 m/s (d) 14 m/s
25. A particle is moving along a circular path with a uniform
18. A bucket tied at the end of a 1.6 m long string is whirled in speed. Through what angle does its angular velocity
a vertical circle with constant speed. What should be the change when it completes half of the circular path?
minimum speed so that the water from the bucket does not
(a) 0 (b) 45
spill when the bucket is at the highest position?
(c) 180 (d) 360
(a) 4 m/sec (b) 6.25 m/sec
(c) 16 m/sec (d) None of these

ANSWER KEY
1. (a) 2. (b) 3. (d) 4. (a) 5. (b) 6. (b) 7. (b) 8. (a) 9.(d) 10. (d) 11.(a)
12.(b) 13.(b) 14. (b) 15.(b) 16. (b) 17. (b) 18. (a) 19. (d) 20. (b) 21. (a) 22. (d) 23. (c)
24. (a) 25. (a)
153

Very Short / Short Answer Questions


1. Two objects having different masses have same momentum. F
F
Which one of them will move faster?
2. At which place on earth, the centripetal force is maximum?
(a) (b)
3. Can a body in linear motion be in equilibrium?
(a) friction will be more in case (a)
4. Why are curved roads generally banked?
(b) friction will be more in case (b)
5. The two ends of a spring-balance are pulled each by a force (c) friction will be equal in both the cases
of 10 kg-wt. What will be the reading of the balance? (d) friction depends on the relations among its dimensions.
6. Why is it easier to maintain the motion than to start it? 19. A block of mass m is placed on a smooth
horizontal surface as shown. The weight
7. What is the angle of friction between two surfaces in contact,
(mg) of the block and normal reaction m
if coefficient of friction is 1/3?
(N) exerted by the surface on the block
8. Explain how proper inflation of tyres saves fuel? (a) form action-reaction pair
9. In a circus in the game of swing, the man falls on a net after (b) balance each other
leaving the swing but he is not injured, why? (c) act in same direction
10. State the laws of limiting friction. Hence define coefficient (d) both (a) and (b)
of friction. 20. Centripetal force :
(a) can change speed of the body.
11. Derive a relation between angle of friction and angle of
(b) is always perpendicular to direction of motion
repose.
(c) is constant for uniform circular motion.
12. Derive the maximum angle by which a cyclist can bend while (d) all of these
negotiating a curved path. 21. When a horse pulls a cart, the horse moves down to
Long Answer Questions (a) horse on the cart.
(b) cart on the horse.
13. State Newtons second law of motion. How does it help to (c) horse on the earth.
measure force. Also state the units of force. (d) earth on the horse.
14. A uniform rod is made to lean between a rough vertical wall 22. The force of action and reaction
and the ground. Show that the least angle at which the rod (a) must be of same nature
can be leaned without slipping is given by (b) must be of different nature
(c) may be of different nature
-1 1 - m1m 2 (d) may not have equal magnitude
q = tan
2m 2 23. A body is moving with uniform velocity, then
(a) no force must be acting on the body.
where m1 and m2 stand for the coefficient of friction between (b) exactly two forces must be acting on the body
(i) the rod and the wall and (ii) the rod and the ground. (c) body is not acted upon by a single force.
15. Name a varying mass system. Derive an expression for (d) the number of forces acting on the body must be even.
velocity of propulsion of a rocket at any instant. 24. The direction of impulse is
16. A force produces an acceleration of 16 m/s2 in a body of mass (a) same as that of the net force
0.5 kg, and an acceleration of 4 m/s2 in another body. If both the (b) opposite to that of the net force
bodies are fastened together, then what is the acceleration (c) same as that of the final velocity
produced by that force? (d) same as that of the initial velocity
17. A gun weighing 10 kg fires a bullet of 30 g with a velocity of 25. A monkey is climbing up a rope, then the tension in the rope
330 m/s. With what velcotiy does the gun recoil? What is (a) must be equal to the force applied by the monkey on
the rope
the resultant momentum of the gun and the bullet before
(b) must be less than the force applied by the monkey on
and after firing? the rope.
Multiple Choice Questions (c) must be greater than the force applied by the monkey
on the rope.
18. A rectangular block is placed on a rough horizontal surface (d) may be equal to, less than or greater the force applied
in two different ways as shown, then by the monkey on the rope.
154

1. A player caught a cricket ball of mass 150 g moving at a rate 8. The upper half of an inclined plane of inclination q is
of 20 m/s. If the catching process is completed in 0.1s, the perfectly smooth while lower half is rough. A block
force of the blow exerted by the ball on the hand of the
player is equal to [CBSE PMT 2001] starting from rest at the top of the plane will again come to
(a) 150 N (b) 3 N rest at the bottom, if the coefficient of friction between the
(c) 30 N (d) 300 N block and lower half of the plane is given by
2. A block of mass m is placed on a smooth wedge of [NEET 2013]
inclination q. The whole system is accelerated horizontally
2
so that the block does not slip on the wedge. The force (a) m = (b) m = 2 tan q
exerted by the wedge on the block (g is acceleration due to tan q
gravity) will be [CBSE PMT 2004] 1
(a) mg/cos q (b) mg cos q (c) m = tan q (d) m =
tan q
(c) mg sin q (d) mg
9. Three blocks with masses m, 2 m and 3 m are connected
3. The coefficient of static friction, ms, between block A of
by strings as shown in the figure. After an upward force F
mass 2 kg and the table as shown in the figure is 0.2. What
would be the maximum mass value of block B so that the is applied on block m, the masses move upward at
two blocks do not move? The string and the pulley are constant speed v. What is the net force on the block of
assumed to be smooth and massless. (g = 10 m/s2) mass 2m?
[CBSE PMT 2004] (g is the acceleration due to gravity) [NEET 2013]
2 kg (a) 2 mg
A
(b) 3 mg
B (c) 6 mg
(a) 0.4 kg (b) 2.0 kg (d) zero
(c) 4.0 kg (d) 0.2 kg
4. A body under the action of a force
r 10. An explosion breaks a rock into three parts in a horizontal
F = 6 i 8 j+10 k,
acquires an acceleration of 1 m/s2. The
plane. Two of them go off at right angles to each other.
mass of this body must be [CBSE-PMT 2009]
(a) 10 kg (b) 20 kg The first part of mass 1 kg moves with a speed of 12 ms1
and the second part of mass 2 kg moves with speed 8 ms1.
(c) 10 2 kg (d) 2 10 kg
If the third part flies off with speed 4 ms1 then its mass is
5. A conveyor belt is moving at a constant speed of 2m/s. A box
is gently dropped on it. The coefficient of friction between [NEET 2013]
them is = 0.5. The distance that the box will move relative to (a) 5 kg (b) 7 kg
belt before coming to rest on it taking g = 10 ms2, is
(c) 17 kg (d) 3 kg
[CBSE-PMT 2011 M]
(a) 1.2 m (b) 0.6 m (c) zero (d) 0.4 m 11. If a body looses half of its velocity on penetrating 3 cm in a
wooden block, then how much will it penetrate more before
6. A body of mass M hits normally a rigid wall with velocity V
and bounces back with the same velocity. The impulse coming to rest? [AIEEE 2002]
experienced by the body is [CBSE-PMT 2011 S] (a) 1 cm (b) 2 cm
(a) MV (b) 1.5 MV (c) 2 MV (d) zero (c) 3 cm (d) 4 cm.
7. A person of mass 60 kg is inside a lift of mass 940 kg and 12. Speeds of two identical cars are u and 4u at a specific instant.
presses the button on control panel. The lift starts moving
upwards with an acceleration 1.0 m/s2. If g = 10 ms2, the The ratio of the respective distances in which the two cars
tension in the supporting cable is [CBSE-PMT 2011 M] are stopped from that instant is [AIEEE 2002]
(a) 8600 N (b) 9680 N (a) 1 : 1 (b) 1 : 4
(c) 11000 N (d) 1200 N (c) 1 : 8 (d) 1 : 16.
155
13. A light string passing over a smooth light pulley connects 19. A horizontal force of 10 N is necessary to just hold a block
two blocks of masses m1 and m 2 (vertically). If the stationary against a wall. The coefficient of friction between
acceleration of the system is g/8, then the ratio of the masses the block and the wall is 0.2. The weight of the block is
is [AIEEE 2002] [AIEEE 2003]
(a) 8 : 1 (b) 9 : 7 (a) 2 N
(c) 4 : 3 (d) 5 : 3.
(b) 100 N
14. A spring balance is attached to the ceiling of a lift. A man
hangs his bag on the spring and the spring reads 49 N, (c) 50 N 10 N
when the lift is stationary. If the lift moves downward with
(d) 20 N
an acceleration of 5m / s 2 , the reading of the spring balance
will be [AIEEE 2003] 20. Two masses m1 = 5kg and m 2 = 4.8kg tied to a string are
(a) 24 N (b) 74 N hanging over a light frictionless pulley. What is the
(c) 15 N (d) 49 N acceleration of the masses when left free to move ?
15. Three forces start acting simultaneously on a particle moving ( g = 9 .8 m / s 2 ) [AIEEE 2004]
r
with velocity, v . These forces are represented in magnitude
(a) 5 m / s2
and direction by the three sides of a triangle ABC (as
shown). The particle will now move with velocity (b) 9.8 m / s 2
[AIEEE 2003]
(c) 0.2 m / s 2
C
(d) 4.8 m / s 2

21. A block rests on a rough inclined plane making an angle of


30 with the horizontal. The coefficient of static friction
A B between the block and the plane is 0.8. If the frictional force
on the block is 10 N, the mass of the block (in kg) is
r
(a) less than v (take g = 10 m / s 2 ) [AIEEE 2004]
r (a) 1.6 (b) 4.0
(b) greater than v
r (c) 2.0 (d) 2.5
(c) v in the direction of the largest force BC
22. A given object takes n times as much time to slide down a
(d) vr , remaining unchanged 45 rough incline as it takes to slide down a perfectly smooth
45 incline. The coefficient of kinetic friction between the
16. A marble block of mass 2 kg lying on ice when given a object and incline is given by [AIEEE 2005]
velocity of 6 m/s is stopped by friction in 10 s. Then the
1 1
coefficient of friction is (Take g = 10 ms2) [AIEEE 2003] (a) 1 - 2 (b)
n 1- n 2
(a) 0.06 (b) 0.03
(c) 0.04 (d) 0.01 1 1
(c) 1 - (d)
4
17. A rocket with a lift-off mass 3.5 10 kg is blasted upwards n2 1- n2
with an initial acceleration of 10m/s2. Then the initial thrust 23. Two fixed frictionless inclined planes making an angle 30
of the blast is [AIEEE 2003] and 60 with the vertical are shown in the figure. Two blocks
(a) 3.5 105 N (b) 7.0 105 N A and B are placed on the two planes. What is the relative
(c) 14.0 105 N (d) 1.75 105 N vertical acceleration of A with respect to B ? [AIEEE 2010]
18. A light spring balance hangs from the hook of the other
light spring balance and a block of mass M kg hangs from A
the former one. Then the true statement about the scale B
reading is [AIEEE 2003]
(a) the scale of the lower one reads M kg and of the upper
one zero
(b) the reading of the two scales can be anything but the
sum of the reading will be M kg
60 30
(c) both the scales read M/2 kg each
(d) both the scales read M kg each
156
(a) 4.9 ms2 in horizontal direction 28. A force F is applied to a block of mass 2 3 kg as shown in
(b) 9.8 ms2 in vertical direction the diagram. What should be the maximum value of force so
(c) Zero that the block does not move ? [IIT JEE 2003]
(d) 4.9 ms2 in vertical direction
F
24. Two cars of masses m1 and m2 are moving in circles of radii
r1 and r2, respectively. Their speeds are such that they make
60 1
complete circles in the same time t. The ratio of their m=
2 3
centripetal acceleration is [AIEEE 2012]
(a) m1r1 : m2r2 (b) m1 : m2
(a) 10 N (b) 20 N
(c) r1 : r2 (d) 1 : 1
(c) 30 N (d) 40 N
25. A string of negligible mass going over a clamped pulley of
29. Two particles of mass m each are tied at the ends of a light
mass m supports a block of mass M as shown in the figure.
string of length 2a. The whole system is kept on a frictionless
The force on the pulley by the clamp is given by horizontal surface with the string held tight so that each
[IIT JEE 2001 S] mass is at a distance 'a' from the center P (as shown in the
m figure). Now, the mid-point of the string is pulled vertically
upwards with a small but constant force F. As a result, the
particles move towards each other on the surface. The
magnitude of acceleration, when the separation between
M them becomes 2x, is [IIT JEE 2004]

F
(a) 2Mg (b) 2mg

(c) [(M + m)2 + m 2 ]g (d) [(M + m)2 + M 2 ]g m m


P
26. The pulleys and strings shown in the figure are smooth and a a
of negligible mass. For the system to remain in equilibrium.
The angle q should be [IIT JEE 2001] F a F x
(a) (b)
2m a 2 , x 2 2m a , x 2
2

F x F a 2 , x2
(c) (d)
2m a 2m x
q
30. The string between blocks of mass m and 2m is massless
2m and inextensible. The system is suspended by a massless
m m spring as shown. If the string is cut, find the magnitudes of
accelerations of mass 2m and m (immediately after cutting).
(a) 0 (b) 30 [IIT JEE 2006]
(c) 45 (d) 60 (a) g, g
27. An insect crawls up a hemispherical surface very slowly, g
(fig). The coefficient of friction between the insect and the (b) g,
2
surface is 1/3. If the line joining the center of the
hemispherical surface to the insect makes an angle a with g
the vertical, the max. possible value of a is given by (c) ,g
2 2m
[IIT JEE 2001 S]
g g
(d) , m
2 2
a 31. A block of base 10 cm 10 cm and height 15 cm is kept on an
inclined plane. The coefficient of friction between them is
3. The inclination q of this inclined plane from the
(a) cot a = 3 (b) sec a = 3 horizontal plane is gradually increased from 0. Then
(c) cosec a = 3 (d) None [IIT-JEE 2009]
157
(a) at q = 30, the block will start sliding down the plane slides along the track without toppling and a frictional force acts
(b) the block will remain at rest on the plane up to certain q on it in the direction opposite to the instantaneous velocity. The
and then it will topple work done in overcoming the friction up to the point Q, as shown
in the figure below, is 150 J.
(c) at q = 60, the block will start sliding down the plane
(Take the acceleration due to gravity, g = 10 ms-2)
and continue to do so at higher angles
y
(d) at q = 60, the block will start sliding down the plane
and on further increasing q, it will topple at certain q
32. A block of mass m is on an inclined plane of angle q. The R P
30
coefficient of friction between the block and the plane is m
and tan q > m. The block is held stationary by applying a
Q R
force P parallel to the plane. The direction of force pointing
up the plane is taken to be positive. As P is varied from
x
P1 = mg (sin q m cos q ) to P2 = mg(sin q + m cos q), the
O
frictional force f versus P graph will look like 33. The magnitude of the normal reaction that acts on the block
[IIT-JEE 2010] at the point Q is [JEE Adv. 2013]
(a) 7.5 N (b) 8.6 N
f (c) 11.5 N (d) 22.5 N
f 34. The speed of the block when it reaches the point Q is
[JEE Adv. 2013]
(a) (b) (a) 5 ms1 (b) 10 ms1
(c) 10 3 ms -1 (d) 20 ms1
35. A bob of mass m, suspended by a string of length l1, is
given a minimum velocity required to complete a full circle in
f f the vertical plane. At the highest point, it collides elastically
with another bob of mass m suspended by a string of length
l2, which is initially at rest. Both the strings are mass-less
(c) (d) and inextensible. If the second bob, after collision acquires
the minimum speed required to complete a full circle in the
l1
vertical plane, the ratio is [JEE Adv. 2013]
l2
Paragraphs for Questions 33 and 34 (a) 3 (b) 5
A small block of mass 1 kg is released from rest at the top of a (c) 6 (d) 8
rough track. The track is a circular arc of radius 40 m. The block

1. An object of mass 10 kg moves at a constant speed of 2. A solid sphere of 2 kg is suspended from a horizontal beam
10 ms1. A constant force, that acts for 4 sec on the object, by two supporting wires as shown in fig. Tension in each
gives it a speed of 2 ms1 in opposite direction. The force wire is approximately (g = 10 ms2)
acting on the object is (a) 30 N 30 30
(a) 3 N (b) 30 N
T T
(c) 3 N (d) 30 N (b) 20 N

(c) 10 N

(d) 5 N mg
158
3. A body of mass 4 kg moving on a horizontal surface with an 11. A constant force F = m2g/2 is applied on the block of mass
initial velocity of 6 ms1 comes to rest after 3 seconds. If m1 as shown in fig. The string and the pulley are light and
one wants to keep the body moving on the same surface the surface of the table is smooth. The acceleration of m1 is
with the velocity of 6 ms1, the force required is
(a) Zero (b) 4 N m1
(c) 8 N (d) 16 N F
4. A toy gun consists of a spring and a rubber dart of mass 16
g. When compressed by 4 cm and released, it projects the
dart to a height of 2 m. If compressed by 6 cm, the height m2
achieved is
(a) 3 m (b) 4 m m2g
(a) towards right
(c) 4.5 m (d) 6 m 2 (m1 + m 2 )
5. A player stops a football weighting 0.5 kg which comes m 2g
flying towards him with a velocity of 10m/s. If the impact (b) towards left
2 (m1 - m 2 )
lasts for 1/50th sec. and the ball bounces back with a
velocity of 15 m/s, then the average force involved is m 2g
(a) 250 N (b) 1250 N (c) towards right
2 (m 2 - m1 )
(c) 500 N (d) 625 N
6. A car travelling at a speed of 30 km/h is brought to a halt in m 2g
(d) towards left
4 m by applying brakes. If the same car is travelling at 60 km/h, 2 (m 2 - m1 )
it can be brought to halt with the same braking power in 12. Consider the system shown in fig. The pulley and the string
(a) 8 m (b) 16 m are light and all the surfaces are frictionless. The tension in
(c) 24 m (d) 32 m the string is (take g = 10 m/s2)
7. A uniform rope of length L resting on a frictionless horizontal
surface is pulled at one end by a force F. What is the tension
in the rope at a distance l from the end where the force is 1 kg
applied.
1 kg
(a) F (b) F (1 + l/L)
(c) F/2 (d) F (1 l/L) (a) 0 N (b) 1 N
8. A machine gun has a mass 5 kg. It fires 50 gram bullets at (c) 2 N (d) 5 N
the rate of 30 bullets per minute at a speed of 400 ms1. 13. The elevator shown in fig. is descending with an
What force is required to keep the gun in position? acceleration of 2 m/s2. The mass of the block A = 0.5 kg.
(a) 10 N (b) 5 N The force exerted by the block A on block B is
(c) 15 N (d) 30 N
9. A force time graph for the motion of a body is shown in Fig.
(a) 2 N
Change in linear momentum between 0 and 8s is

F (N) (b) 4 N 2 m/s


2

(c) 6 N A
1
0 x (d) 8 N
2 4 6 7 8 B
t (s)
14. Two blocks of masses 2 kg and 1 kg are placed on a smooth
2 horizontal table in contact with each other. A horizontal
force of 3 newton is applied on the first so that the block
moves with a constant acceleration. The force between the
(a) zero (b) 4 N-s blocks would be
(c) 8 Ns (d) None of these (a) 3 newton (b) 2 newton
10. Fig. shows a uniform rod of length 30 cm having a mass of
(c) 1 newton (d) zero
3.0 kg. The strings shown in the figure are pulled by
constant forces of 20 N and 32 N. All the surfaces are smooth 15. A cart of mass M has a block of mass m attached to it as
and the strings and pulleys are light. The force exerted by shown in fig. The coefficient of friction between the block
20 cm part of the rod on the 10 cm part is and the cart is m. What is the minimum acceleration of the
cart so that the block m does not fall?
10
cm 20 cm
(a) mg
(b) g/m M m
20 N 32 N
(c) m/g
(a) 20 N (b) 24 N
(c) 32 N (d) 52 N (d) M mg/m
159
16. A rocket has a mass of 100 kg. Ninety percent of this is fuel. A
It ejects fuel vapors at the rate of 1 kg/sec with a velocity of
500 m/sec relative to the rocket. It is supposed that the rocket 5 kg
is outside the gravitational field. The initial upthrust on the
rocket when it just starts moving upwards is
(a) zero (b) 500 newton 5 kg B
(c) 1000 newton (d) 2000 newton
17. A particle of mass m moving eastward with a speed v (a) 49 N (b) Zero
collides with another particle of the same mass moving (c) 36.75 N (d) 2.45 N
northward with the same speed v. The two particles 25. A 40 kg slab rests on frictionless floor as shown in fig. A 10
coalesce on collision. The new particle of mass 2m will move kg block rests on the top of the slab. The static coefficient
in the north-external direction with a velocity : of friction between the block and slab is 0.60 while the
(a) v/2 (b) 2v kinetic friction is 0.40. The 10 kg block is acted upon by a
horizontal force of 100 N. If g = 9.8 m/s2, the resulting
(c) v/ 2 (d) None of these
acceleration of the slab will be:
18. A spring is compressed between two toy carts of mass m 1
and m2. When the toy carts are released, the springs exert 100 N
equal and opposite average forces for the same time on
each toy cart. If v1 and v2 are the velocities of the toy carts
and there is no friction between the toy carts and the
ground, then : 40 kg
(a) v1/v2 = m1/m2 (b) v1/v2 = m2/m1 No friction
(c) v1/v2 = m2/m1 (d) v1/v2 = m1/m2
(a) 0.98 m/s2 (b) 1.47 m/s2
19. A man weighing 80 kg is standing on a trolley weighing 320
kg. The trolley is resting on frictionless horizontal rails. If (c) 1.52 m/s2 (d) 6.1 m/s2
the man starts walking on the trolley along the rails at a 26. Two blocks are connected over a massless pulley as shown in
speed of one metre per second, then after 4 seconds, his fig. The mass of block A is 10 kg and the coefficient of kinetic
displacement relative to the ground will be : friction is 0.2. Block A slides down the incline at constant
speed. The mass of block B in kg is:
(a) 5 metres (b) 4.8 metres
(c) 3.2 metres (d) 3.0 metres
20. Starting from rest, a body slides down a 45 inclined plane
in twice the time it takes to slide down the same distance in A
the absence of friction. The coefficient of friction between
the body and the inclined plane is: 30 B
(a) 0.33 (b) 0.25 (a) 3.5 (b) 3.3
(c) 0.75 (d) 0.80 (c) 3.0 (d) 2.5
21. A ball of mass 0.5 kg moving with a velocity of 2 m/sec 27. Two trolleys of mass m and 3m are connected by a spring.
strikes a wall normally and bounces back with the same They were compressed and released at once, they move off
speed. If the time of contact between the ball and the wall is in opposite direction and come to rest after covering a
one millisecond, the average force exerted by the wall on distance S1, S2 respectively. Assuming the coefficient of
the ball is : friction to be uniform, ratio of distances S1 : S2 is :
(a) 2000 newton (b) 1000 newton (a) 1 : 9 (b) 1 : 3
(c) 5000 newton (d) 125 newton (c) 3 : 1 (d) 9 : 1
28. A particle of mass 10 kg is moving in a straight line. If its
22. The mass of the lift is 100 kg which is hanging on the string.
The tension in the string, when the lift is moving with displacement, x with time t is given by x = (t3 2t 10) m,
constant velocity, is (g = 9.8 m/sec2) then the force acting on it at the end of 4 seconds is
(a) 24 N (b) 240 N
(a) 100 newton (b) 980 newton
(c) 300 N (d) 1200 N
(c) 1000 newton (d) None of these 29. A particle of mass m is moving with velocity v1, it is given
23. In the question , the tension in the strings, when the lift is an impulse such that the velocity becomes v2 . Then
accelerating up with an acceleration 1 m/sec2, is magnitude of impulse is equal to
(a) 100 newton (b) 980 newton r r r r
(a) m( v2 - v1 ) (b) m( v1 - v 2 )
(c) 1080 newton (d) 880 newton r r r r
24. A block of mass 5 kg resting on a horizontal surface is (c) m (v 2 - v1 ) (d) 0.5m(v 2 - v1 )
connected by a cord, passing over a light frictionless pulley 30. A force of 10 N acts on a body of mass 20 kg for 10 seconds.
to a hanging block of mass 5 kg. The coefficient of kinetic Change in its momentum is
friction between the block and the surface is 0.5. Tension in (a) 5 kg m/s (b) 100 kg m/s
the cord is : (g = 9.8 m/sec2) (c) 200 kg m/s (d) 1000 kg m/s
160
31. When forces F1, F2, F3 are acting on a particle of mass m
such that F2 and F3 are mutually perpendicular, then the
particle remains stationary. If the force F1 is now removed
then the acceleration of the particle is (a) (b)
v v
(a) F1/m (b) F2F3/mF1
(c) (F2 F3)/m (d) F2/m
32. One end of massless rope, which passes over a massless
and frictionless pulley P is tied to a hook C while the
other end is free. Maximum tension that the rope can
(c) (d) v
bear is 360 N. With what value of maximum safe v
acceleration (in ms 2 ) can a man of 60 kg moves
downwards on the rope? [Take g = 10 ms2] 38. A particle starts sliding down a frictionless inclined plane.
If Sn is the distance traveled by it from time t = n 1 sec to
P
t = n sec, the ratio Sn/Sn+1 is
C
2n - 1 2n + 1
(a) (b)
2n + 1 2n
(a) 16 (b) 6 2n 2n + 1
(c) 4 (d) 8 (c) (d)
2n + 1 2n - 1
33. Two mass m and 2m are attached with each other by a rope
passing over a frictionless and massless pulley. If the pulley 39. A block is kept on a inclined plane of inclination q of length l .
is accelerated upwards with an acceleration a, what is the The velocity of particle at the bottom of inclined is (the
value of T? coefficient of friction is m )
g+a g -a
(a) (b) (a) [2gl(m cos q - sin q)]1 / 2 (b) 2gl(sin q - m cos q)
3 3

4 m (g + a ) m (g - a ) (c) 2gl(sin q + m cos q) (d) 2gl(cos q + m sin q)


(c) (d)
3 3 40. Blocks A and B of masses 15 kg and 10 kg, respectively, are
34. A uniform chain of length 2 m is kept on a table such that a connected by a light cable passing over a frictionless pulley
length of 60 cm hangs freely from the edge of the table. The as shown below. Approximately what is the acceleration
total mass of the chain is 4 kg. What is the work done in experienced by the system?
pulling the entire chain on the table ?
(a) 2.0 m/s2
(a) 12 J (b) 3.6 J
(c) 7.2 J (d) 1200 J (b) 3.3 m/s2
35. A mass is hanging on a spring balance which is kept in a
lift. The lift ascends. The spring balance will show in its (c) 4.9 m/s2
readings B
(a) an increase (d) 9.8 m/s2 A
(b) a decrease
41. A 50 kg ice skater, initially at rest, throws a 0.15 kg snowball
(c) no change
with a speed of 35 m/s. What is the approximate recoil speed
(d) a change depending on its velocity
of the skater?
36. A block of mass 0.1kg is held against a wall applying a
(a) 0.10 m/s (b) 0.20 m/s
horizontal force of 5 N on the block. If the coefficient of
friction between the block and the wall is 0.5, the magnitude (c) 0.70 m/s (d) 1.4 m/s
of the frictional force acting on the block is: 42. Block A is moving with acceleration A along a frictionless
(a) 2.5 N (b) 0.98 N horizontal surface. When a second block, B is placed on
(c) 4.9 N (d) 0.49 N top of Block A the acceleration of the combined blocks
37. A small block is shot into each of the four tracks as shown drops to 1/5 the original value. What is the ratio of the mass
below. Each of the tracks rises to the same height. The of A to the mass of B?
speed with which the block enters the track is the same in (a) 5 : 1 (b) 1 : 4
all cases. At the highest point of the track, the normal (c) 3 : 1 (d) 2 : 1
reaction is maximum in
161
43. A force F is used to raise a 4-kg mass M from the ground to 49. The coefficient of friction between two surfaces is 0.2. The
a height of 5 m. angle of friction is
(a) sin -1 (0.2) (b) cos -1 (0.2)

(c)tan -1 (0.1) (d) cot -1 (5)


60 r
50. A force F = 8i - 6 j - 10k newton produces an acceleration
of 1 ms2 in a body. The mass of the body is
F
M (a) 10 kg (b) 10 2 kg
What is the work done by the force F? (Note : sin 60 = 0.87; (c) 10 3 kg (d) 200 kg
cos 60 = 0.50. Ignore friction and the weights of the pulleys) 51. A block of mass 4 kg rests on an inclined plane. The
(a) 50 J (b) 100 J inclination to the plane is gradually increased. It is found
(c) 174 J (d) 200 J that when the inclination is 3 in 5, the block just begins to
44. A 5000 kg rocket is set for vertical firing. The exhaust speed slidedown the plane. The coefficient of friction between
is 800 m/s. To give an initial upward acceleration of 20 m/s2, the block and the plane is
the amount of gas ejected per second to supply the needed
(a) 0.4 (b) 0.6
thrust will be (Take g = 10 m/s2)
(a) 127.5 kg/s (b) 137.5 kg/s (c) 0.8 (d) 0.75.
(c) 155.5 kg/s (d) 187.5 kg/s 52. A bird is in a wire cage which is hanging from a spring
45. A bullet is fired from a gun. The force on the bullet is given balance . In the first case, the bird sits in the cage and in the
by F = 600 2 105 t second case, the bird flies about inside the cage. The
Where, F is in newtons and t in seconds. The force on the reading in the spring balance is
bullet becomes zero as soon as it leaves the barrel. What is (a) more in the first case
the average impulse imparted to the bullet? (b) less in first case
(a) 1.8 N-s (b) Zero (c) unchanged
(c) 9 N-s (d) 0.9 N-s (d) zero in second case.
46. A 4000 kg lift is accelerating upwards. The tension in the 53. A rider on a horse back falls forward when the horse
-2 suddenly stops. This is due to
supporting cable is 48000 N. If g = 10m s then the
acceleration of the lift is (a) inertia of horse
-2 -2 (b) inertia of rider
(a) 1 m s (b) 2 m s
(c) large weight of the horse
-2 -2
(c) 4 m s (d) 6 m s (d) losing of the balance
47. A 0.1 kg block suspended from a massless string is moved 54. A ball of mass m is thrown vertically upwards. What is the
rate at which the momentum of the ball changes?
first vertically up with an acceleration of 5 ms -2 and then
(a) Zero
moved vertically down with an acceleration of 5 ms -2 . If (b) mg
T1 and T2 are the respective tensions in the two cases, (c) Infinity
then (d) Data is not sufficient.
(a) T2 > T1 55. A body of mass 1 kg moving with a uniform velocity of

(b) T1 - T2 = 1 N, if g = 10 ms -2 1 ms -1 . If the value of g is 5 ms -2 , then the force acting on


(c) T1 - T2 = 1kg f the frictionless horizontal surface on which the body is
moving is
(d) T1 - T2 = 9.8N, if g = 9.8 ms-2 (a) 5 N (b) 1 N
48. A rifle man, who together with his rifle has a mass of 100 kg, (c) 0 N (d) 10N
stands on a smooth surface and fires 10 shots horizontally. 56. A body of mass 2 kg is placed on a horizontal surface
Each bullet has a mass 10 g and a muzzle velocity of 800
having kinetic friction 0.4 and static friction 0.5. If the force
ms1. The velocity which the rifle man attains after firing
applied on the body is 2.5 N, then the frictional force acting
10 shots is
on the body will be [g = 10 ms2]
-1
(a) 8 ms -1 (b) 0.8 ms (a) 8 N (b) 10 N
(c) 0.08 ms -1 (d) 0.8 ms -1 (c) 20 N (d) 2.5 N
162
57. A bag of sand of mass m is suspended by a rope. A bullet 62. A triangular block of mass M with angles 30, 60, and 90
rests with its 3090 side on a horizontal table. A cubical
m block of mass m rests on the 6030 side. The acceleration
of mass is fired at it with a velocity v and gets
20 which M must have relative to the table to keep m stationary
relative to the triangular block assuming frictionless
embedded into it. The velocity of the bag finally is contact is
v 20v g
(a) g (b)
(a) 21 (b)
20 21 2
g g
(c) (d)
v v 3 5
(c) (d)
20 21 63. A block of mass m on a rough horizontal surface is acted
upon by two forces as shown in figure. For equilibrium of
58. For the arrangement shown in the Figure the tension in block the coefficient of friction between block and surface
the string is [Given : tan -1 (0.8) = 39 ] is

F2
q

m = 1 kg
F1 m
m = 0.8
39
F1 + F2 sin q F1 cos q + F2
(a) (b)
(a) 6 N (b) 6.4 N mg + F2 cos q mg - F2 sin q
(c) 0.4 N (d) zero. F1 + F2 cos q F1 sin q - F2
59. A 1 kg block and a 0.5 kg block move together on a (c) (d)
mg + F2 sin q mg - F2 cos q
horizontal frictionless surface . Each block exerts a force of 64. A weight W rests on a rough horizontal plane. If the angle
6 N on the other. The block move with a uniform of friction be q , the least force that will move the body
acceleration of along the plane will be
(a) W cos q (b) W cot q
a
F 1 kg 0.5 kg (c) W tan q (d) W sin q
65. A trailer of mass 1000 kg is towed by means of a rope
attached to a car moving at a steady speed along a level
(a) 3 ms -2 (b) 6 ms -2 road. The tension in the rope is 400 N. The car starts to
accelerate steadily. If the tension in the rope is now 1650 N,
(c) 9 ms -2 (d) 12 ms -2 with what acceleration is the trailer moving ?
60. A body of mass 32 kg is suspended by a spring balance (a) 1.75 ms2 (b) 0.75 ms2
(c) 2.5 ms 2 (d) 1.25 ms2
from the roof of a vertically operating lift and going
downward from rest. At the instant the lift has covered 20 66. A rocket of mass 5000 kg is to be projected vertically upward.
The gases are exhausted vertically downwards with
m and 50 m, the spring balance showed 30 kg and 36 kg
velocity 1000 ms2 with respect to the rocket. What is the
respectively. Then the velocity of the lift is minimum rate of burning the fuel so as to just lift the rocket
(a) decreasing at 20 m, and increasing at 50 m upwards against gravitational attraction ?
(b) increasing at 20m and decreasing at 50 m (a) 49 kg s1 (b) 147 kg s1
(c) continuously decreasing at a steady rate throughout (c) 98 kg s1 (d) 196 kg s1
67. In the figure a smooth pulley of negligible weight is
the journey suspended by a spring balance. Weight of 1 kg f and
(d) constantly increasing at constant rate throughout the 5 kg f are attached to the opposite ends of a string passing
journey. over the pulley and move with acceleration because of
61. An object at rest in space suddenly explodes into three gravity, During their motion, the spring balance reads a
parts of same mass. The momentum of the two parts are weight of

2pi and pj . The momentum of the third part (a) 6 kg f

(a) will have a magnitude p 3 (b) less then 6 kg f


(b) will have a magnitude p 5
(c) more than 6 kg f
(c) will have a magnitude p
(d) will have a magnitude 2p. (d) may be more or less then 6 kg f 1 kg
5 kg
163
68. A particle moves so that its acceleration is always twice its 74. In the system shown in figure, the pulley is smooth and
velocity. If its initial velocity is 0.1 ms1, its velocity after it massless, the string has a total mass 5g, and the two
has gone 0.1 m is suspended blocks have masses 25 g and 15 g. The system
(a) 0.3 ms1 (b) 0.7 ms1 is released from state l = 0 and is studied upto stage l' = 0
(c) 1.2 ms 1 (d) 3.6 ms1 During the process, the acceleration of block A will be
69. An object is resting at the bottom of two strings which are
inclined at an angle of 120 with each other. Each string g
can withstand a tension of 20N. The maximum weight of (a) constant at
9
the object that can be supported without breaking the l l'
string is g A
(b) constant at 25 g
(a) 5 N (b) 10 N 4 B
(c) 20 N (d) 40 N (c) increasing by factor of 3 15 g
70. On a smooth plane surface (figure) two block A and B are (d) increasing by factor of 2
accelerated up by applying a force 15 N on A. If mass of B 75. A horizontal force F is applied on back of mass m placed on
is twice that of A, the force on B is a rough inclined plane of inclination q . The normal reaction
(a) 30 N (b) 15 N N is
(c) 10 N (d) 5 N
F
15 N A B

71. A 10 kg stone is suspended with a rope of breaking strength


30 kg-wt. The minimum time in which the stone can be
(a) mg cos q (b) mg sin q
raised through a height 10 m starting from rest is (Take
g = 10 N / kg) (c) mg cos q - Fcos q (d) mg cos q + F sin q
76. The coefficient of friction between the rubber tyres and the
(a) 0.5 s (b) 1.0 s road way is 0.25. The maximum speed with which a car can
(c) be driven round a curve of radius 20 m without skidding is
2/3 s (d) 2 s
(g = 9.8 m/s2)
-1 (a) 5 m/s (b) 7 m/s
72. A ball of mass 0.4 kg thrown up in air with velocity 30 ms
(c) 10 m/s (d) 14 m/s
reaches the highest point in 2.5 second . The air resistance
77. A bucket tied at the end of a 1.6 m long string is whirled in
encountered by the ball during upward motion is
a vertical circle with constant speed. What should be the
(a) 0.88 N (b) 8800N minimum speed so that the water from the bucket does not
(c) 300 dyne (d) 300 N. spill when the bucket is at the highest position?
73. A plate of mass M is placed on a horizontal of frictionless (a) 4 m/sec (b) 6.25 m/sec
surface (see figure), and a body of mass m is placed on this (c) 16 m/sec (d) None of the above
plate. The coefficient of dynamic friction between this body 78. A cane filled with water is revolved in a vertical circle of
and the plate is m . If a force 2 m mg is applied to the body radius 4 meter and the water just does not fall down. The
of mass m along the horizontal, the acceleration of the time period of revolution will be
plate will be (a) 1 sec (b) 10 sec
(c) 8 sec (d) 4 sec
m
2m mg 79. A circular road of radius r in which maximum velocity is v,
M has angle of banking
v2 rg
(a) tan -1 (b) tan -1
rg v2
mm mm
(a) g (b) g
M ( M + m)
v rg
(c) tan -1 (d) tan -1
2mm rg v
(c) 2mm (d) g.
g
M ( M + m)
164
80. A small sphere is attached to a cord and rotates in a vertical 85. The linear momentum p of a body moving in one dimension
circle about a point O. If the average speed of the sphere is varieswith timeaccording to theequating P= a + bt2 where
increased, the cord is most likely to break at the orientation a and b are positive constants. The net force acting on the
body is
when the mass is at
(a) proportional to t2
A
(b) a constant
m
(c) proportional to t
l
C D (d) inversely proportional to t
O 86. Three blocks of masses m1, m2 and m3 are connected by
massless strings, as shown, on a frictionless table. They
B are pulled with a force T3 = 40 N. If m1 = 10 kg, m2 = 6 kg and
m3 = 4kg, the tension T2 will be
(a) bottom point B (b) the point C
T1 T2 T3
(c) the point D (d) top point A M1 M2 M3
81. A person with his hand in his pocket is skating on ice at the
rate of 10m/s and describes a circle of radius 50 m. What is (a) 20 N (b) 40 N
his inclination to vertical : (g = 10 m/sec2) (c) 10 N (d) 32 N
(a) tan1() (b) tan1 (1/5) 87. In an explosion, a body breaks up into two pieces of unequal
(c) tan1 (3/5) (d) tan 1(1/10) masses. In this
82. When the road is dry and the coefficient of the friction is m, (a) both parts will have numerically equal momentum
(b) lighter part will have more momentum
the maximum speed of a car in a circular path is 10 ms 1. If (c) heavier part will have more momentum
m (d) both parts will have equal kinetic energy
the road becomes wet and m' = , what is the maximum 88. A body of mass 1.0 kg is falling with an acceleration of 10
2
m/sec2. Its apparent weight will be (g = 10 m/sec2)
speed permitted? (a) 1.0 kg wt (b) 2.0 kg wt
(a) 5 ms1 (b) 10 ms1 (c) 0.5 kg wt (d) zero
89. A ball of mass 400 gm is dropped from a height of 5 m. A
(c) 10 2 ms -1 (d) 5 2 ms -1 boy on the ground hits the ball vertically upwards with a
83. The minimum velocity (in ms-1) with which a car driver bat with an average force of 100 newton so that it attains a
vertical height of 20 m. The time for which the ball remains
must traverse a flat curve of radius 150 m and coefficient of
in contact with the bat is (g = 10 m/s2)
friction 0.6 to avoid skidding is (a) 0.12 s (b) 0.08 s
(a) 60 (b) 30 (c) 0.04 s (d) 12 s
(c) 15 (d) 25 90. Block A of weight 100 kg rests on a block B and is tied with
horizontal string to the wall at C. Block B is of
84. Two pulley arrangements of figure given are identical. The 200 kg. The coefficient of friction between A and B is 0.25
mass of the rope is negligible. In fig (a), the mass m is lifted
1
by attaching a mass 2m to the other end of the rope. In fig and that between B and surface is . The horizontal force
(b), m is lifted up by pulling the other end of the rope with 3
a constant downward force F = 2mg. The acceleration of m F necessary to move the block B should be (g = 10 m/s2)
in the two cases are respectively
A
C
B F

(a) 1050 N (b) 1450 N


(c) 1050 N (d) 1250 N
91. An open topped rail road car of mass M has an initial
velocity v0 along a straight horizontal frictionless track. It
suddenly starts raising at time t = 0. The rain drops fall
m 2m m F = 2 mg vertically with velocity u and add a mass m kg/sec of water.
(a) (b) The velocity of car after t second will be (assuming that it is
not completely filled with water)
u mv0
(a) 3g, g (b) g / 3 , g (a) v0 + m (b)
M M + mt
(c) g / 3 , 2g (d) g, g / 3 Mv0 + ut mut
(c) (d) v 0 +
M + ut M + ut
165
92. A ball mass m falls vertically to the ground from a height h1 99. A round uniform body of radius R, mass M and moment of
and rebounds to a height h 2. The change in momentum of inertia I rolls down (without slipping) an inclined plane
the ball of striking the ground is making an angle q with the horizontal. Then its acceleration
(a) m 2g(h1 + h 2 ) (b) n 2g(m1 + m 2 ) is

(a) g sin q (b) g sin q


(c) mg(h1 - h 2 ) (d) m( 2gh1 - 2gh 2 )
93. In the given figure, the pulley is assumed massless and 1, MR / I
2 1 I / MR 2
frictionless. If the friction force on the object of mass m is f, g sin q
then its acceleration in terms of the force F will be equal to (c) (d) g sin q
1 MR 2 / I 1, I / MR 2
100. A block of mass m is connected to another block of mass M
by a spring (massless) of spring constant k. The block are
F kept on a smooth horizontal plane. Initially the blocks are at
m
rest and the spring is unstretched. Then a constant force F
starts acting on the block of mass M to pull it. Find the force
F of the block of mass m.
(a) (F - f ) / m (b) - f / m
2
MF mF
(c) F/m (d) None of these (a) (b)
94. A smooth block is released at rest on a 45 incline and then (m + M) M
slides a distance d. The time taken to slide is n times as
(M + m)F mF
much to slide on rough incline than on a smooth incline. (c) (d)
The coefficient of friction is m (m + M)
101. A body of mass m = 3.513 kg is moving along the x-axis with
1 1 a speed of 5.00 ms1. The magnitude of its momentum is
(a) m k = 1- (b) m k = 1-
n2 n2 recorded as
(a) 17.6 kg ms1 (b) 17.565 kg ms1
1 1 (c) 17.56 kg ms 1 (d) 17.57 kg ms1
(c) m s = 1- (d) m s = 1-
n 2
n2
Directions for Qs. (102 to 109) : Read the following passage(s)
95. The upper half of an inclined plane with inclination f is carefully and answer the questions that follows:
perfectly smooth while the lower half is rough. A body
starting from rest at the top will again come to rest at the PASSAGE I
bottom if the coefficient of friction for the lower half is given A student performs a series of experiments to determine the
by coefficient of static friction and the coefficient of kinetic friction
(a) 2 cos f (b) 2 sin f between a large crate and the floor. The magnitude of the force of
(c) tan f (d) 2 tan f static friction is always less than or equal to msN. where ms denotes
the coefficient of static friction, and N denotes the normal force
96. A particle of mass 0.3 kg subject to a force F = kx with k =
exerted by the floor on the crate:
15 N/m . What will be its initial acceleration if it is released
from a point 20 cm away from the origin ? fs m s N
(a) 15 m/s2 (b) 3 m/s2 Static friction exists only when the crate is not sliding across the
(c) 10 m/s2 (d) 5 m/s2 floor.
97. A block is kept on a frictionless inclined surface with angle
of inclination a . The incline is given an acceleration a to The force of kinetic friction is given by f k = m k N
keep the block stationary. Then a is equal to where mk denotes the coefficient of kinetic friction. Kinetic friction
exists only when the crate is sliding across the floor.
The create has mass 100 kg. In this situation, the normal force
points upward.
Experiment 1
The student pushes horizontally (rightward) on the crate. and
a gradually increases the strength of this push force. The create
does not begin to move until the push force reaches 400 N.
Experiment 2
(a) g cosec a (b) g / tan a
The student applies a constant horizontal (rightward) push force
(c) g tan a (d) g
98. Consider a car moving on a straight road with a speed of 100 for 1.0 seconds and measures how far the crate moves during
m/s . The distance at which car can be stopped is [k = 0.5] that time interval. In each trial. the crate starts at rest, and the
(a) 1000 m (b) 800 m student stops pushing after the 1.0-second interval. The following
(c) 400 m (d) 100 m table summarizes the results.
166

Trial Push force (N) Distance (m)


Y
1 500 1.5 arad fs
2 600 1.5
3 700 2.0
102. The coefficient of static friction between the crate and (b) X
floor is approximately:
(a) 0.25 (b) 0.40 mg
(c) 0.40 (d) 4.0 R
103. In experiment 1. when the rightward push force was 50 N.
the crate didnt move. Why didnt it move? Y
(a) The push force was weaker than the frictional force
on the crate. arad
fs
(b) The push force had the same strength as the
gravitational force on the crate.
(c) X R
(c) The push force was weaker than the frictional force
on the crate.
(d) The push force had the same strength as the mg
frictional force on the crate.
104. The coefficient of kinetic friction between the crate and Y
the floor is approximately:
(a) 0.20 (b) 0.30 arad
fs
(c) 0.40 (d) 0.50
105. In trial 3, what is the crates speed at the moment the R
(d) X
student stops pushing it?
(a) 1.0 m/s (b) 2.0 m/s
(c) 3.0 m/s (d) 4.0 m/s mg
106. For trial 3, which of the following graphs best shows the
positions of the crate as a function of time? The student 108. What minimum coefficient of static friction is required if
first starts pushing the crate at time t = 0. the person on the ride is not to slide downward to the new
position of the floor?
(a) 0.28 (b) 0.50
position

position

(c) 0.39 (d) 0.01


(a) (b) 109. Mark the correct statement/s
0 0
1 time(s) 1 time(s) (a) Under same conditions a heavy man will fall down
(b) Answer of the above question will depend upon mass
of the passenger
position

position

(c) Answer of the above question is independent of


(c) (d) mass of passenger
0 0
1 time(s) 1 time(s) (d) For smaller ms larger v must be kept to maintain the
man in equilibrium
PASSAGE 2 Directions for Qs. (110 to 112) : Each question contains
On the ride "spindletop" at the amusement park Six Flags Over STATEMENT-1 and STATEMENT-2. Choose the correct answer
Texas, people stood against the inner wall of a hallow vertical (ONLY ONE option is correct ) from the following-
cylinder with radius 2.5 m. The cylinder started to rotate, and (a) Statement -1 is false, Statement-2 is true
when it reached a constant rotation rate of 0.60 rev/s, the floor on (b) Statement -1 is true, Statement-2 is true; Statement -2 is a
which people were standing dropped about 0.5 m. The people correct explanation for Statement-1
remained pinned against the wall.
(c) Statement -1 is true, Statement-2 is true; Statement -2 is not
107. Point out the best possible force diagram for a person on
this side after the floor has dropped (fs is force of friction a correct explanation for Statement-1
and R is Reaction) (d) Statement -1 is true, Statement-2 is false
Y
R 110. Statement -1 : The work done in bringing a body down
fs
from the top to the base along a
frictionless incline plane is the same as
the work done in bringing it down the
(a) X arad vartical side.
mg Statement -2 : The gravitational force on the body
along the inclined plane is the same as
that along the vertical side.
167
111. Statement -1 : On a rainy day, it is difficult to drive a height if the air resistance for each be
car or bus at high speed. the same then both the bodies will reach
Statement -2 : The value of coefficient of friction is the earth simultaneously.
lowered due to wetting of the surface. Statement -2 : For same air resistance, acceleration of
112. Statement -1 : The two bodies of masses M and m (M both the bodies will be same.
> m) are allowed to fall from the same

Exercise 5.1 2. (b) When elevator goes up, then equation of motion is
R mg = ma R = m(g+a) i.e.,
1. (c) When the swimmer push some water in backward apparent weight > real weight
direction, then he get some momentum in forward
When elevator goes down, then equation of motion is
direction from water & starts to swim. This is according
mg R = ma R = m(ga) i.e.,
to Newton third Law. (action-reaction force)
2. (d) Newton first law of motion is also called law of inertia as apparent weight < real weight
it defines inertia. 3. (d) An inertial frame of reference is one in which law of
3. (c) The body will continue accelerating until the resultant inertia holds good i.e. Newtons laws of motion are
force acting on the body becomes zero. applicable equally. If earth is revolving around the sun
4. (b) See Newtons first law of motion according to which or earth is rotating about its axis, then forces are acting
the tendency of a body to continue in its state of rest or on the earth and hence there will be acceleration of earth
of uniform motion in a straight line, is called law of inertia. due to these forces. That is why earth can not be an
5. (c) He can come at shore by making use of Newtons third inertial frame of reference.
law. In this case man push the ice backward & ice reacts 4. (c) When an elevator cabin falls down, it is accelerated down
back to the man in forward direction due to friction
with respect to earth i.e. man standing on earth.
between ice & man. If friction is very small between him
5. (d) The observations will be true if both the frames are inertial
& the ice, then he come out from this pond only by
or non inertial.
taking very small steps.
6. (c) The gun applied a force F12 on the bullet in forward 6. (d) In this case
direction & according to Newtons third law bullet applies R = mg a = 0
a reaction force on gun F21 in backward direction. But So elevator may at rest or in uniform motion (either up or
the recoil speed of gun is very low in comparison to down)
bullet due to large mass. 7. (c) R = apparent weight = m(ga)
dp 8. (d) R = apparent weight = m(g+a)
7. (b) F =
dt 9. (b) By spitting or sneezing we get a momentum in opposite
9. (b) When jet plane flies, it ejects gases in back ward direction direction which will help us in getting off the plane. In all
at very high velocity. From Newtons third law, these other cases we will slip on ice as there is no friction.
gases provides the momentum to jet plane in forward
10. (b) R Fsinq
direction plus compensates the force of gravity. R F pulling
pushing Block
10. (c) From Newtons second law if SFi = 0 then the body is f Block q Fcosq
W
in translational equilibrium. Here f = mR=mW f
Pushing W
Here f = m(W-Fsinq)
Exercise 5.2 Pulling

1. (a) When tension in the cable is equal to the weight of Since we required less force in pulling in comparison of
cable, the system is in equilibrium. It means the system pushing it. Hence pulling is easier then pushing.
is at rest or moving with uniform velocity.
168

Exercise 5.3 R

1. (b) From Newtons second law, the total external applied


5. (c) F
force on the body is equal to the time rate change of f
momentum of the body. mg
dp m(v 2 - v1 )
F= = here v1 = 0, v2 = v So equation of motion is F f = ma
dt t
where f = mR = mMg = m 309.8 = 294m
(mv) Since initial velocity of the body is zero & its velocity
so F = = mvn
1/ n increases from zero to 4 m/sec in 2 seconds, so
2. (b) It works on the principle of conservation of linear acceleration of the body is
momentum. v = u + at
3. (a) For a given mass P V. If the momentum is constant 4 = 0 + a 2 or a = 2 m/sec2
then its velocity must be constant. So Ff = 302
mG v G 1 5 200 m 294 = 60
4. (d) mGvG = mBvB v B = =
mB 10 10 3 140
m= = 0.476
= 500 ms1 294
6. (c) q = Angle of repose \ m = tan q.
Exercise 5.4
7. (c) When the men push the rough surface on walking, then
1. (d) mstatic > mkinetic > mrolling surface (from Newton third Law) applies reaction force
in forward direction. It occurs because there is friction
2. (a) Static friction is a self adjusting force in magnitude and between men & surface. If surface is frictionless (such
direction. as ice), then it is very difficult to move on it.
3. (d) In case (a) In case (b)
8. (d) When brakes are on, the wheels of the cycle will slide on
the road instead of rolling there. It means the sliding
N mN N
F2 friction will come into play instead of rolling friction.
The value of sliding friction is more than that of rolling
F1
friction.
q mg q mg
mg Cos q mN q Sin mg Cos q S in
q q q Exercise 5.5
mg mg
1. (a) If the outside rail is h units higher than inside of rail track
as shown in figure then
mg sin q = F1 mN N cos q = mg....................(1)
N = mg cos q
\ mg sin q + m mg cos q = F1
N Ncosq Train
In second case (b)
q q
mN + F2 = mg sin q sin A
N
h
m mg cos q F2 = mg sin q O q
mg B
or F2 = mg sin q mmg cos q inside
but F1 = 2F2
trrack
Rail

outside
therefore mg sin q + m mg cos q
= 2(mg sin q m mg cos q)
mv 2
mg sin q = 3 m mg cos q N sin q = ...................(ii)
r
or tan q = 3m or q = tan1 (3m)
v2
4. (c) ms > mk > mr & tan q = ....................(iii)
rg
169
Where q is angle of banking of rail track, N is normal G
reaction exerted by rail track on rail. N1 N2
It is clear from the equation (i) & (ii) that N cosq balance B h
f1
the weight of the train & N sinq provide the necessary A
inner wheel f2 mg outer wheel
centripetal force to turn. of car of car
If width of track is l (OB) & h (AB) be height of outside
2a
of track from the inside, then

h v2 v2l gra
tan q = = or h = .................(iv) v max =
l rg rg h
So it is clear from the above analysis that if we increase where r is radius of the path followed by car for turn & 2a
the height of track from inside by h metre then resultant is distance between two wheels of car (i.e., AB)
force on rail is provided by railway track & whose r r r r
6. (b) p B - p A = m(v B - v A ) = mv (j + j)
direction is inwards.
2. (b) It means that car which is moving on a horizontal road & = 2m v j = 2 kg m/s ....(i)
the necessary centripetal force, which is provided by
friction (between car & road) is not sufficient. r r m v2 m v2
If m is friction between car and road, then max speed of FB - FA = (- i) - ( + i)
R R
safely turn on horizontal road is determined from figure.
2 m v2
= (- i) = 8 N .....(ii)
R
N
v
car of Divide (ii) by (i), = = 4 rad/s.
f mass m R
mg 7. (b) Due to centrifugal force, the inner wheel will be left up
when car is taking a circular turn. Due to this, the reaction
on outer wheel is more than that on inner wheel.
8. (a) Here, u = 10 3 m/s, t = ? ; q = 30
N = mg ...(i)
At t = 0, the vertical component of velocity is zero,
mv 2 hence horizontal component of velocity at t = 0 is 103 m/
f= ...(ii) sec
r
At time t, v sin 30 = 0 +gt & v cos 30 =103
Where f is frictional force between road & car, N is the
normal reaction exerted by road on the car. We know gt
or tan q =
that v
f = m s N = m s mg ......(iii)
\ gt = u tan q = 10 3 tan 30 = 10
where ms is static friction
t = 10/g = 10/10 = 1 sec.
so from eq (ii) & (iii) we have
mv 2 tower
t=0 sec
m s mg v 2 m s rg or v m s rg
r 103

& v max = m s rg t=? 30


v
If the speed of car is greater than vmax at that road, then
it will be thrown out from road i.e., skidding. 9. (d) Centripetal acceleration = acceleration due to gravity
4. (a) The car over turn, when reaction on inner wheel of car is
v2
zero, i.e., first the inner wheel of car leaves the ground = g or v = R g
R
(where G is C.G of car, h is height of C.G from the ground,
f1 & f2 are frictional force exerted by ground on inner & 1
10. (d) mv 2 = m g l or v = ( 2 g l)
outer wheel respectively). The max. speed for no over 2
turning is
170
11. (a) The velocity should be such that the centripetal 18. (a) Since water does not fall down, therefore the velocity of
acceleration is equal to the acceleration due to gravity revolution should be just sufficient to provide centripetal
acceleration at the top of vertical circle. So,
v2
= g or v = g R v = ( g r ) = {10 (1 .6)} = (16 ) = 4 m/sec.
R

m v2 19. (d) The speed at the highest point must be v rg


12. (b) = mg or v = g r
r Now v = rw = r (2p / T )
13. (b) See fig. The body will lose contact when centripetal

acceleration becomes equal to the component of \ r(2p / T) rg or T 2 p r 2 p r
acceleration due to gravity along the radius. rg g

u=0 \ T = 2p 4 = 4 sec
9.8
P rh
r v 2
v 2 72 1000
q h 20. (b) tan q = = / 20 10 = 2
rg 3600

O g 1
21. (a) Since kinetic energy K = mv 2 = as 2 (given)
2
Velocity at P, v = 2 g ( r - h ) (Q v 2 - u 2 = 2gx )
mv2 2as 2
Centripetal acceleration will be v2/r. It should be equal so centripetal force FC = =
R R
to the component of g along PO. Hence
22. (d) tan q = v2 / rg, tan q = H / 1.5, r = 200 m, b = 1.5 m
v2 2g ( r - h ) h v = 36 km/hour = 36 (5/18) = 10 m/s.
= g cos q or = g Putting these values, we get H = 0.075 m.
r r r
23. (c) The tension T1 at the topmost point is given by
2r
Solving we get, h = m v12
3 T1 = - mg
20
14. (b) Let particle of mass m move in circle of radius r with Centrifugal force acting outward while weight acting
uniform speed v. Then L is defined as downward.

1 L2 m v 22
L = mvr or mv 2 = The tension T2 at the lowest point T2 = + mg
2 2mr 2 20
Centrifugal force and weight (both) acting downward
v
m v 22 - m v12
T2 - T1 = + 2mg
20
r
v12 = v 22 - 2 g h or v 22 - v12 = 2 g ( 40) = 80 g

80 m g
15. (b) Centripetal acceleration = v2/r. It is perpendicular to the \ T2 - T1 = + 2mg = 6mg
20
increase in speed a which is tangential.
24. (a) The velocity at the lowest point is given by v = ( 2 g r )
2 2
\ Resultant acceleration =
v 2
+ a ( ) m v2
r Further, T - mg = (at lowest point)
r
16. (b) Thrust = weight centripetal force
m v2 m (2 g r)
2
\ T = mg + = mg +
17. (b) m mg = m v / r or v = m g r r r
=m g+2mg=3mg
or v = ( 0 .25 9 .8 20 ) = 7 m / s
171
25. (a) There is no change in the angular velocity, when speed 5. (d) Frictional force on the box f = mmg
is constant.
\ Acceleration in the box
Exercise 1 : NCERT Based Questions
a = mg = 5 ms2
1. Object with smaller mass. v2 = u2 + 2as
2. At the pole
0 = 22 + 2 (5) s
3. Yes
4. To help in providing centripetal force needed for motion of 2
s= w.r.t. belt
vehicles on the curved road. 5
5. 10 kg-wt. distance = 0.4 m
6. As the dynamic friction is less than the force of limiting 6. (c) Impulse experienced by the body
friction.
= change in momentum
7. 30.
16. 3.2 m/s2 = MV (MV)
17. 0.99 m/s zero = 2MV.
18. (c) 19. (b) 20. (b)
21. (d) 22. (a) 23. (c) 7. (c) a=1
24. (a) 25. (a)
Exercise 2 : PAST Competition MCQs

m( v - u ) 0.15(0 - 20) m = 1000 kg


1. (c) F= = = 30 N
t 0. 1
2. (a) N = m a sin q + mg cos q ......(1)
also m g sin q = m a cos q ......(2)
from (2) a = g tan q Total mass = (60 + 940) kg = 1000 kg
2 Let T be the tension in the supporting cable, then
sin q
\ N = mg + mg cos q ,
cos q T 1000g = 1000 1

mg T = 1000 11 = 11000 N
or N =
cos q 8. (b)

m
ac
os S/2
h
q N oot S/2 sin q
ma Sm
q
sq q S/2
g co
m m ugh S/2 sin q
q
cos mg g
sin Ro
ma q
q
For upper half of inclined plane
3. (a) mBg = ms mAg {Q mAg = ms mAg}
v2 = u2 + 2a S/2 = 2 (g sin q) S/2 = gS sin q
mB = ms mA
For lower half of inclined plane
or mB = 0.2 2 = 0.4 kg
r 0 = u2 + 2 g (sin q m cos q) S/2
4. (c) F = 6 i 8 j+10 k,

| F |= 36 + 64 + 100 = 10 2 N ( Q F = Fx2 + Fy2 + Fz2 ) gS sin q = gS ( sinq m cos q)


2 sin q = m cos q
a = 1 ms2
Q F = ma 2 sin q
m= = 2 tan q
10 2 cos q
\ m= = 10 2 kg
1
172
In IInd case when the body comes to rest, final velocity

v
9. (d) = 0, initial velocity =
2
F T T'
m v
2
mg v v2
mg Again, (0)2 = 2. .s; or s = 1cm
m 2m 3m 2 8
2m
T T' T"
2mg So the extra penetration will be 1 cm.
mg 2mg 3mg
3m 12. (d) Use u2 = 2as. a is same for both cases.
s1 = u2/2a ; s2 = 16u2/2a = 16s1 s1 : s2 = 1 : 16.

6 mg m1 - m 2 1 m - m2
13. (b) a = g ; = 1
m1 + m 2 8 m1 + m 2

From figure m1 : m2 = 9 : 7.
F = 6 mg,
As speed is constant, acceleration a = 0 14. (a) 49
Mass = = 5 kg
\ 6 mg = 6ma = 0, F = 6 mg 9.8
\ T = 5 mg , T = 3 mg When lift is moving downward
T = 0
Apparent weight = 5(9.8 5) = 5 4.8 = 24 N
Fnet on block of mass 2 m
15. (d) According to triangle law of forces, the resultant force
= T T' 2 mg = 0
is zero.
ALTERNATE :
In presence of zero external force, there is no change in
Q v = constant
so, a = 0, Hence, Fnet = ma = 0 velocity.

6
10. (a) 16. (a) a = mg = [using v = u + at]
y 10
2 kg m2
8 m/sec 6 6
Presultant m= = = 0.06
10 g 10 10
12 m/sec
m1 17. (a) Thrust = Mass Acceleration
c x
/se 1 kg = 3.5 104 10 = 3.5 105 N
4m
3
m 18. (d) Since both springs are very light i.e., mass less. Hence
tension T is same in both spring & it is equal to Mg.
||
Presultant = 122 + 162
||
= 144 + 256 = 20
m3v3 = 20 (momentum of third part) T srping 1

20 ||
or, m3 = = 5 kg
4
|| ||
11. (a) Let the initial velocity of the body be v. Hence the final T srping 2
velocity = v/2
v
2 Mg
2 2
Applying v = u 2as = v2 2.a.3
2 19. (a) mg = mF = 0.2 10 = 2N
a = v2/8
173
26. (c) If T is the tension in the string , Then
m1 - m 2
20. (c) Acceleration = g T = mg (For outer masses)
m + m 1 2
2T cos q = 2mg (For inner mass)
( 5 - 4 . 8) 9 .8 2
= m / s 2 = 0. 2 m / s Eliminating T, we get
( 5 + 4 . 8)
2(mg) cos q = 2mg
fs
or cos q = 1/ 2 q = 45
N
27. (a) From F.B.D of insect
21. (c)
mg f = mg sin a m k R
For max, value of a
fmax= mkR = mg sin a & R = mg cos a
mg sinq = fs ( for body to be at rest)
1
m 10 sin 30 = 10 cot a = =3 (Q m k = 1 / 3)
mk
m = 2.0 kg

2s 2s mkR R a
22. (a) We have =n
g(sin q - m cos q) gsin q
mg mgsina
cosa
mg
2s 2s n 2
=
g (sin q - m cos q) g sin q Alternatively :
As is clear from Fig.
1
here q = 45 = n 2 or m = (1 - 1/ n 2 ) F = mg sin a, R = mg cos a
1-m
F
23. (a) mg sin q = ma = tan a i.e. m = tan a = 1
R 3
\ a = g sin q
where a is along the inclined plane \ cot a = 3
\ vertical component of acceleration is g sin2 q 28. (d) Balancing vertical forces, we have
\ relative vertical acceleration of A with respect to B is N = Fsin 60 + mg
g
g (sin 2 60 - sin 2 30] = = 4.9 m/s2 For the block not to move, we must have
2
in vertical direction Fcos 60 = mN
24. (c) As their period of revolution is same, so it their angualr
i.e., Fcos 60 = m(Fsin 60 + mg)
speed. Centripetal acceleration is circular path,

a = w2 r . 1 1 3
F = F + (2 3)(10)N
a1 w 2 r1 r1 2 2 3 2
Thus, = =
a2 w 2 r2 r2
1 1
25. (d) The force on the pulley by the clamp will balance the or F - = 10N
resultant of the tension forces acting on the pulley 2 4
(= mg) and the weight of the pulley and block or F = 4 10N = 40N
[= (M + m)g]. Hence force on the pulley by the clamp 29. (b) The acceleration of mass m is due to the force T cos q
= [{(M + m)g}2 + M 2g 2 ] \ T cos q = ma

T cos q
= [(M + m)2 + M 2 ]g a=
m
174
F
T T N
f
O 15 c
m
Also, F = 2T sin q a q
cm
a 10
T sin q T sin q P
F M
T= q T cos q w
2 sin q
x x
From (i) and (ii) q

F cos q F F x In DPOM,
a = = =
2sin q m 2m tan q 2m PM 5cm 2
a - x2
2
q = tan q = = =
OM 7.5cm 3
For this, q < 60. From this we can conclude that the
a2 - x2 block will topple at lesser angle of inclination. Thus
Q tan q =
x the block will remain at rest on the plane up to a certain
anlgle q and then it will topple.
30. (c) By equilibrium of mass m, 32. (a) As tan q > m, the block has a tendency to move down
T' = mg ....(i) the incline. Therfore a force P is applied upwards along
By equilibrium of mass 2m, the incline.
Here, at equilibrium P + f = mg sin q
T = 2mg T' ....(ii)
f = mg sin q - P
From (i) & (ii),
T = 2mg mg = mg ....(iii) N
f

P q
mg sin q mg cos q

mg
T T
2m 2m q
2mg
T' 2mg
T' Now as P increases, f decreases linearly with respect
m m to P.
When P = mg sin q, f = 0.
mg mg When P is increased further, the block has a
Situation 1 Situation 2 tendency to move upwards along the incline.
When the string is cut :
N
For mass m :
Fnet = mam mg = ma m a m = g
P q
For mass 2m : mg cos q
mg sin q f
Fnet = 2ma2m 2mg - T = 2ma 2m
mg
g q
2 mg - mg = 2 ma 2m a 2 m =
2
31. (b) For the block to slide, the angle of inclination should Therefore the frictional force acts downwards along
be equal to the angle of repose, i.e., the incline.
Here, at equilibrium P = f + mg sin q
tan -1 m = tan -1 3 = 60. Therefore, option (a) is
\ f = P mg sin q
wrong.
For the block to topple, the condition of the block will Now as P increases, f increases linearly w.r.t P.
be as shown in the figure. This is represented by graph (a) .
175
1 2
33. (c) in P.E. of the dart ( = mgh) i.e. kx = mgh
2
30 1
\ k ( 4 ) 2 = 16 g 200 ....(i)
40 sin 30 2
40m 1
and k (6) 2 = 16 g h ...(ii)
2
N On solving, (i) and (ii), we get h = 450 cm = 4.5 m.
5. (d) Here m = 0.5 kg ; u = 10 m/s ;
mg sin 60 t = 1/50 s ; v = + 15 ms1
v Force = m (v u)/t = 0.5 (10 + 15) 50 = 625 N
60
6. (b) As, (1/2)m v2 = Fs
mg cos 60 mg
1 1
So m (30) 2 = F 4 and m ( 60 ) 2 = F s
2 2
mv2 \ s/4 = (60)2 / (30)2 = 4 or s = 4 4 = 16 m.
N mg cos 60 =
r 7. (d) Let n be the mass per unit length of rope. Therefore,
mass of rope = nL.
mv2 Acceleration in the rope due to force F will be
\ N = mg cos 60 + ...(1) a = F/nL.
r
Mass of rope of length (L l) will be n (L l).
Loss in P.E. = mg 40 sin 30 = 200 J
Therefore, tension in the rope of length (L l), is equal
Work done in over coming friction = 150 J
to pulling force on it
\ K.E. possessed by the particle = 50 J = n (L l) a = n (L l) F/nL = F (1 l/L)
1 change in momentum
\ mv 2 = 50J 8. (a) Force required =
2 time taken
\ mv2 = 100 J ...(2)
(50 10 -3 30) 400 - (5 0)
1 100 = = 10 N
From (1) and (2), N = 1 10 + = 5 + 2.5 = 7.5 N 60
2 40 9. (a) Change in momentum = Force time = Area which the
(a) is the correct option. force-time curve encloses with time axis.
34. (a) From (2), mv2 = 100
10. (b) 10cm 20cm
\ v = 10 ms1 20N(F1) F F 32N(F2)
(b) is the correct option. l1 l2
35. (b) Velocity at the highest point of bob tied to string l1 is L
acquired by the bob tied to string l2 due to elastic It is clear F2 > F1, so rod moves in right direction with
head-on collision of equal masses an acceleration a, whereas a is given by
Therefore gl1 = 5gl 2 (F2F1)= mLa................(i)
l where m is mass of rod per unit length.
\ 1 =5
l2 Now consider the motion of length l1 from first end,
then
Exercise 3 : Conceptual & Applied MCQs F F1 = ml1a..................(ii)
1. (b) Here u = 10 ms1, v = 2 ms1, Dividing eq (ii) by (i), we get
F - F1 l
t = 4 s, a = ? = 1 or F = (F - F ) l1 + F
F2 - F1 L 2 1
L
1
v - u - 2 - 10
Using a = = = -3 m / s 2 here l1 = 10 cm., L = 30 cm., F1 = 20 N, F2 = 32N
t 4
so F = 24 N
\ Force, F = ma = 10(3) = 30 N
2. (b) 2 T cos 60 = mg 11. (a) Let a be the acceleration of mass m 2 in the downward
direction. Then
or T = mg = 210 = 20 N.
T m2 (g/2) = m1 a ....(i)
3. (c) Acceleration, a = v - u = 0 - 6 = -2 ms -2 and m2 g T = m2 a ....(ii)
t 3 Adding eqs. (1) and (2), we get
Force = ma = 42 = 8 N (m1 + m2) a = m2g m2 (g/2) = m2 g/2
4. (c) If k is the spring factor, then P.E. of the spring
m2 g
\a=
compressed by distance x = kx 2 will equal to gain
1 2 (m 1 + m 2 )
2
176
12. (d) See fig. 18. (c) Applying law of conservation of linear momentum
T m1 v v m
m1v1 + m2v2 = 0, = - 2 or 1 = - 2
T m2 v1 v2 m1
1 kg
T 19. (c) Displacement of the man on the trolley
1 kg = 1 4 = 4m
Now applying conservation of linear momentum
From figure, 1 g T = 1 a ...(i)
and T = 1 a ....(ii) 1
80 1 + 400 v = 0 or v = - m/sec.
From eqs. (i) and (ii), we get 5
1g 1a = 1a or 2a = g The distance travelled by the trolley
\ a = (g/2) = (10/2) = 5 m/s2 = 0.2 4 = 0.8 m.
So, T = ma = 1 5 = 5 N (In opposite direction to the man.)
13. (b) R = mg ma = 0.5 10 0.5 2 = 5 1 = 4 N Thus, the relative displacement of the man with the
14. (c) See fig. Let F be the force between the blocks and a their ground = (4 0.8) = 3.2 m.
common acceleration. Then for 2 kg block, 20. (c) In presence of friction a = (g sinq mg cos q)
\ Time taken to slide down the plane
3N 2 kg
1 kg 2s 2s
F t1 = =
F a g (sin q - m cos q)
3F=2a ...(1)
2s
for 1 kg block, F = 1 a = a ....(2) In absence of friction t 2 =
g sin q
\ 3 F = 2 F or 3 F = 3 or F = 1 newton
15. (b) See fig. Given : t 1 = 2 t 2

mN 2s 2s 4
\ t12 = 4t 22 or =
g (sin q - m cos q) g sin q
ma N sin q = 4 sinq 4m cos q
3 3
mg m= tan q = = 0.75 (since q = 45)
4 4

If a = acceleration of the cart, then N = ma mv - (- mv ) 2mv 2 0.5 2


21. (a) F= = = = 2 103 N
\ mN = mg or m ma = mg or a = g/m t t 10 -3
Dm 22. (b) T = m (g + a) = 100 (9.8 + 0) = 980 N
16. (b) Initial thrust on the rocket = v rel
Dt 23. (c) T = m(g+a) = 100(9.8+1) = 1080N
= 500 1 = 500 N
24. (c) For block A, T mN = 5a and N = 5g
Dm
where = rate of ejection of fuel. N
Dt
17. (c) p1 = mv northwards, p2 = mv eastwards
A a
N
T
5 kg
m v
mN
W E a
T
v
m 5 kg B
S 5 kg

Let p = momentum after collision. Then,


r r r r
p = p1 + p 2 or p = (mv) 2 + (mv) 2 for block B, 5g T = 5a
v T = 36.75N, a = 2.45 m/sec2
2 mv = mv 2 or v = m/sec
2
177
25. (a) Force on the slab (m = 40 kg) = reaction of frictional 31. (a)
force on the upper block F3

100 N
10 kg F2
mk 10 g m
40 kg F1

\ 40a = mk 10 g or a = 0.98 m/sec2 The formula for force is given by F1 = ma


26. (b) Considering the equilibrium of B F1
mBg + T = mBa Acceleration of the particle a = ,
m
Since the block A slides down with constant speed. because F1 is equal to the vector sum of F2 & F3.
a = 0. 32. (c)
Therefore T = mBg
Considering the equilibrium of A, we get P
10a = 10g sin 30 T mN
where N = 10g cos 30 C T

a a
N mg
T

A mN T a mg - T = ma
30
g sin B
10 60 10 - 360
10g cos30 =a
60
mBg
10g
a = 4 ms-2
10 33. (c) The equations of motion are
\ 10 a = g - T - m 10 g cos 30
2 2 mg T = 2ma
but a = 0, T = mBg T mg = ma T = 4ma & a = g/3 so T = 4mg/3
0 .2 3 If pulley is accelerated upwards with an accleration a,
0 = 5g - m B g - 10 g then tension in string is
2
mB = 3.268 3.3 kg 4m
T= (g + a )
v1 3
27. (d) mv1 + 3mv2 = 0 or = -3
v2 4
34. (b) Mass of over hanging chain m = (0.6) kg
2
1
Now mv 12 = F. S1 = m. mg . S1 Let at the surface PE = 0
2
C.M. of hanging part = 0.3 m below the table
1 4
(3m ) v 22 = F. S 2 = m. 3 mg . S 2 U i = - m gx = - 0.6 10 0.30
2 2
S1 v12 9 DU = m 'gx = 3.6J = Work done in putting the entire
or = =
S 2 v 22 1 chain on the table

28. (b) m = 10 kg, x = (t3 2t 10) m 35. (a) Let acceleration of lift = a and
let reaction at spring balance = R
dx d 2x R
= n = 3t 2 - 2 = a = 6t
dt dt 2
At the end of 4 seconds, a = 6 4 = 24 m/s2
F = ma = 10 24 = 240 N
r r
29. (a) Impulse = change in momentum = m v 2 - m v1
30. (b) Change in momentum = F t mg
= 10 10 = 100 Ns or 100 kg. m/s Applying Newtons law
178

R mg = ma R = m(g + a ) ( l = length of incline)


thus net weight increases, or, v = 2gl (sin q - m cos q)
So reading of spring balance increases.
36. (b) The magnitude of the frictional force f has to balance 40. (a) Two external forces, FA and FB, act on the system and
the weight 0.98 N acting downwards. move in opposite direction. Lets arbitrarily assume that
the downward direction is positive and that FA provides
m=0.5 downward motion while FB provides upward motion.
FA = (+15 kg) (9.8 m/s2) = 147 N
and FB = (10 kg)(9.8 m/s2) = 98 N
5N Ftotal = FA + FB = 147 N + (98 N) = 49 N
5N The total mass that must be set in motion is
5N
15 kg + 10 kg = 25kg
Since Ftotal = m total a , a = Ftotal / mtotal
0.19.8
=0.98 N = 49 N / 25 kg @ 2 m/s2
41. (a) Momentum is always conserved. Since the skater and
Therefore the frictional force = 0.98 N snowball are initially at rest, the initial momentum is
zero. Therefore, the final momentum after the toss must
mv ' 2
37. (a) At the highest point of the track, N + mg = also be zero.
r
Pskater + Psnowball = 0
or m skater v skater + m snowball v snowball = 0
mg v skater = -m snowball vsnowball / m skater
N
= -(0.15kg)(35m / s) =
where r is the radius of curvature at that point and v is - 0.10m / s
(50kg)
the speed of the block at that point.
The negative sign indicates that the momenta of the
mv ' 2 skater and the snowball are in opposite directions.
Now N = - mg
r 42. (b) Apply Newtons second law
N will be maximum when r is minimum (v is the same for FA = FAB, therefore :
all cases). Of the given tracks, (a) has the smallest radius mA aA = (mA + mB)aAB and aAB = aA / 5
of curvature at the highest point. Therefore : mA aA = (mA + mB)aA/5 which reduces to
a 4 mA = mB or 1 : 4
38. (a) Sn = (2 n - 1)
2 43. (d) Work is the product of force and distance. The easiest
way to calculate the work in this pulley problem is to
a multiply thenet force or the weight mg by the distance
Sn +1 = (2n + 1)
2 it is raised: 4 kg x 10 m/s2 x 5 m = 200 J.
44. (d) Given : Mass of rocket (m) = 5000 Kg
Sn 2n - 1
= Exhaust speed (v) = 800 m/s
Sn +1 2n + 1
Acceleration of rocket (a) = 20 m/s2
39. (b) From the F.B.D. Gravitational acceleration (g) = 10 m/s2
N = mg cos q We know that upward force
F = ma = mg sin q mN F = m (g + a) = 5000 (10 +20)
= 5000 30 = 150000 N.
a = g (sin q - m cos q)
We also know that amount of gas ejected
N mN dm F 150000
= = = 187.5 kg / s
dt v 800
mg sin q m g cos q
xmg 45. (d) Given F = 600 2 105 t
q The force is zero at time t, given by
Now using, v 2 - u 2 = 2as 0 = 600 2 105 t

or, v 2 = 2 g (sin q - m cos q)l 600


t= = 3 10 3 seconds
2 105
179

t 310 3 q = tan -1 (0.8) = 39


\ Impulse = Fdt = (600 2 105 t) dt
0 0
The given angle of inclination is equal to the angle of
310 3
repose. So the 1 kg block has no tendency to move.
2 105 t 2
= 600t \ mg sin q = force of friction
2
0
T=0
= 600 3 10 3 105 (3 10 3 ) 2 59. (d) For 0.5 kg block, 6 = 0.5 a
60. (b) While moving down, when the lift is accelerating the
= 1.8 0.9 = 0.9Ns weight will be less and when the lift is decelerating the
46. (b) T = m (g + a ) weight will be more.
48000 = 4000(10 + a ) 61. (b) Total momentum = 2pi + pj
-2 Magnitude of total momentum
a = 2 ms
47. (b) T1 = m(g + a ) = 0.1(10 + 5) = 1.5N = ( 2 p) 2 + p 2 = 5p 2 = 5p

T2 = m(g - a ) = 0.1(10 - 5) = 0.5N This must be equal to the momentum of the third part.

T1 - T2 = (1.5 - 0.5) N = 1N
s 30
48. (b) According to law of conservation of momentum, 62. (c) co
ma
100v = -
10
10 800 30 60

1000 30 ma (pseudo force)
sin
ie, v = 0.8 ms1. mg M a
-1
49. (d) Angle of friction = tan 30 90

82 + (-6)2 + (-10)2 ma cos 30 = mg sin 30


50. (b) m= = 10 2kg
1
g
3 \a=
51. (d) sin q = 3
5
5 63. (a) Here, on resolving force F2 and applying the concept
3 of equilibrium
q
4 F2cos q
3 3 F2sin q
\ tan q = m = tan q = = 0.75
4 4
F1 m
52. (a) Based on Newtons third law of motion.
53. (b) Inertia is resistance to change.
f N
54. (b) The time rate of change of momentum is force.
mg
55. (a) Weight of body = m g = 5 N
56. (d) Limiting friction = 0.5 2 10 = 10N N = mg + F2 cos q , and f = N
The applied force is less than force of friction, therefore \ f = m[mg + F2 cos q] (i)
the force of friction is equal to the applied force.
57. (d) Applying law of conservation of momentum Also f = F1 + F2 sin q (ii)
Momentum of bullet = Momentum of sand-bullet From (i) and (ii)
system m[mg + F2 cos q] = F1 + F2 sin q
m m 21 F1 + F2 sin q
v= m + V = mV m=
20 20 20 mg + F2 cos q
58. (d) Here tan q = 0.8 64. (c) f = mW
where q is angle of repose
f = W tan q [\ m = tan q ]
180
65. (d) Here, the force of friction is 400N.
Final net force = ( 25 + 5 ) - 15 g = 15 g
Fnet = (1650 - 400) = 1250N
(acceleration)final = 3 (acceleration)initial
1250 75. (d)
\ a= = 1.25ms - 2
1000
q
os
66. (a)
dm mg
= =
5000 9.8
= 49 kg s -1 Fc
1000
N q
dt vr
F
67. (b) Reading of spring balance F sin q
4 5 1 10 q
4m1m 2
= = kgf sin mg cos q
2T =
m1 + m 2 6 3 mg mg
68. (a) a = 2v (given)
dv From figure N = mg cos q + F sin q
v = 2v
ds
76. (b) m mg = m v 2 / r or v = m g r
or dv = 2ds
or v = ( 0 .25 9 .8 20 ) = 7 m / s
v
77. (a) Since water does not fall down, therefore the velocity
dv = 2[s]0
0.1
= 0.2
of revolution should be just sufficient to provide
0.1
centripetal acceleration at the top of vertical circle. So,
v - 0.1 = 0.2 = 4 m/sec.
v = (g r ) = {10 (1.6)} = (16)
v = 0.3ms -1
78. (d) The speed at the highest point must be v rg
69. (c) If W is the maximum weight, then
W = 2T cos 60 Now v = rw = r (2p / T )
or W = T = 20N
\ r ( 2 p / T ) > rg or T < 2pr < 2p r
15 5 rg g
70. (c) The acceleration of both the blocks = =
3x x
5 \ T = 2p 4 = 4 sec
\ Force on B = 2x = 10 N 9.8
x
71. (b) The maximum acceleration that can be given is a 79. (a) From figure,
\ 30g = 10g + 10a mv2
N sin q = ....... (i)
r
a = 2g = 20ms -2
N cos q = mg ...... (ii)
1 2 Dividing, we get
We know that s = ut + at
2
v2 v2
tan q = or q = tan -1
2s 2 10 rg rg
\t= = =1s
a 20
80. (a) In the case of a body describing a vertical circle,
72. (a) Let the air resistance be F. Then A
mg + F = ma F = m[a - g]
30
Here a = = 12ms - 2 O
2. 5 C D
q T
73. (a) The frictional force acting on M is mg
mmg
\ Acceleration = q Mg cos q
M B
74. (c) Considering the two masses and the rope a system, Mg sin q Mg
then
mn 2 mn 2
Initial net force = [ 25 - (15 + 5)] g = 5g T - mg cos q = T = mg cos q +
l l
181
Tension is maximum when cos q = +1 and velocity is and 2mg - T ' = 0 (4)
maximum
Solving (3) and (4)
Both conditions are satisfied at q = 0 (i.e. at lowest
point B) a'=g
81. (b) Since surface (ice) is frictionless, so the centripetal g
force required for skating will be provided by inclination \ a= and a ' = g
3
of boy with the vertical and that angle is given as
85. (c) Linear momentum, P = a + bt 2
v2
tan = where v is speed of skating & r is radius
rg dP
= 2bt (on differentiation)
of circle in which he moves. dt

82. (d) v max = mgr dP


\ Rate of change of momentum, t
dt
83. (b) The condition to avoid skidding, v = mrg
dP
By 2nd law of motion, F
= 0.6 150 10 = 30 m/s. dt
84. (b) Let a and a' be the accelerations in both cases \ F t
respectively. Then for fig (a), 86. (d) For equilibrium of all 3 masses,
T3 = (m1 + m2 + m3)a or

T3
a=
m1 + m 2 + m3

a For equilibrium of m1 & m2


T T a
T2 = (m1 + m 2 ).a

(m1 + m 2 )T3
or, T2 =
mg 2mg m1 + m 2 + m3
(a)
Given m1 = 10 kg, m2 = 6 kg, m3 = 4 kg, T3 = 40 N

T mg = ma (1) (10 + 6).40


\ T2 = = 32N
and 2mg T = 2ma (2) 10 + 6 + 4

Adding (1) and (2), we get 87. (a) If m1, m2 are masses and u1, u2 are velocity then by
conservation of momentum m 1 u1 + m 2 u2 = 0 or
mg = 3ma
| m1u1 | = | m2 u 2 |
g 88. (d) Apparent weight when mass is falling down is given
\a=
3 by W ' = m(g - a)
For fig (b),
\ W ' = 1 (10 - 10) = 0
89. (a) Velocity of ball after dropping it from a height of
5m

10 m/sec 20 m/sec
a T T a

(using v2 = u2 + 2gh)
F = 2mg v2 = 0 + 2 10 5 v = 10 m/s
mg Velocity gained by ball by force exerted by bat
(b)
0 = u2 2gh
u2 = 2 10 20 or u = 20 m/s
T '- mg = ma ' (3)
Change in momentum = m(u + v)
182
= 0.4 (20 + 10) = 12 kg m/s For sliding the block
T f = force on the block = mass acceleration
DP DP
F= or Dt = T-f
Dt F or acceleration of block = . Put T from (i)
m
12
Dt = = 0.12sec F
100 -f
\ Acceleration = 2
90. (d) F1 = Force of friction between B and A m
= m1m1g
= 0.25 100 g = 25 g newton g sin q - mg cos q
94. (b)
F2 = Force of friction between (A + B) and surface d q
n
= m2 m 2 g = m2 (mass of A and B) g g si d
45 45
1 300 smooth rough
= (100 + 200)g = g = 100g newton
3 3
When surface is When surface is
\ F = F1 + F2 smooth rough
= 25 g + 100 g = 25g = 125 10 N
\ F = 1250 N 1 1 2
d= (g sin q)t 12 , d = (g sin q - mg cos q) t 2
91. (b) The rain drops falling vertically with velocity u do not 2 2
affect the momentum along the horizontal track. A
vector has no component in a perpendicular direction 2d 2d
t1 = , t2 =
Rain drops add to the mass of the car g sin q g sin q - mg cos q
Mass added in t sec = (mt) kg
Momentum is conserved along horizontal track. According to question, t 2 = nt1
Initial mass of car = M
Initial velocity of car = v0 2d 2d
n =
Final velocity of (car + water) = v g sin q g sin q - mg cos q
Mass of (car + water) after time t = (M + mt)
m , applicable here, is coefficient of kinetic friction as
\ final momentum = initial momentum
the block moves over the inclined plane.
(M + mt)v = Mv0
1 1
Mv 0 n= Q cos 45 = sin 45 =
\ v= 1- mk 2
(M + mt)
92. (d) Let v1 = velocity when height of free fall is h 1 1 1
v2 = velocity when height of free rise is h2 n2 = or 1 - mk =
1- mk n2
\ v12 2
= u + 2gh1 for free fall
or 1
or m k = 1-
For free rise after impact on ground n2
95. (d) Acceleration of block while sliding down upper half =
0 = v22 - 2gh 2 or v 22 = 2gh 2
Initial momentum = mv1 g sin f ;
Final momentum = mv2 retardation of block while sliding down lower half =
\ Change in momentum = m(v1 v2) (g sin f - mg cos f)
= m( 2gh 2 - gh 2 ) For the block to come to rest at the bottom, acceleration
93. (b) T = tension is the string in I half = retardation in II half.
\ Applied force F = 2T g sin f = -(g sin f - mg cos f)
T = F/2 (i)
m = 2 tan f
Alternative method : According to work-energy
T
theorem, W = DK = 0
T T F (Since initial and final speeds are zero)
f m
\ Work done by friction + Work done by gravity
For block of mass m, force of friction due to surface f. =0
183
101. (a) Momentum, p = m v
l
i.e., -( mg cos f) + mgl sin f = 0 = (3.513) (5.00) = 17.565 kg m/s
2
= 17.6 (Rounding off to get three significant figures)
PASSAGE 1
or cos f = sin f or = 2 tan f
2 102. (b) Sincefs is always less than or equal to msN, its maximum
96. (c) Mass (m) = 0.3 kg F = m.a = 15 x possible value is
fs max = msN = msmg.
15 -150 (The normal force must equal the crates weight,
a= x= x = - 50 x
0.3 3 because the vertical forces cancel.)

a = 50 0.2 = 10 m / s 2
N
97. (c) From free body diagram, Fpush
f
ma
g cos
N
a a W = mg
a a
mg cosa The crate starts to move when the push force barely
+ ma sina mg mg sin a
exceeds.fs max. This happens when
For block to remain stationary, Fpush 400 N. So, 400N fs Max = msmg
mg sin a = ma cos a a = g tan a = ms(100 kg)(10 m /s2) = ms(1000N).
Therefore, ms 0.40.
98. (a) v 2 - u 2 = 2as or 02 - u 2 = 2(-mk g) s 103. (d) If fs were bigger than Fpush, the crate would accelerate
leftward, because it would feel a net leftward force.
1 Therefore (c) is wrong.
- 100 2 = 2 - 10 s
2 Many students choose C because they calculate fs =
s = 1000 m msmg = 400 N. But thats the maximum possible force
99. (b) This is a standard formula and should be memorized. of static friction. Static friction adjusts itself,
becoming bigger or smaller as needed in order to
g sin q cancel the push force. When the push force is only 50
a=
I N, static friction reduces itself to 50 N. Thats why we
1+
MR 2 write is fs ms N instead of f s = ms N
100. (d) Writing free body-diagrams for m & M, 104. (b) First, use a kinematic equation to find the crates
acceleration during a particular trial. Then, apply
M Newtons 2nd law, Fnet = ma. This reasoning works no
m
k matter which trial you consider. Here, well use trial
F
1 2
N N 1.Since x = n0 t + at , and since the crate begins
a 2
T T M 1 2
m F with no velocity, we get x= at or
2
mg Mg 1
1.0m = a(1.0s)2
2
we get,
T = ma and F T = Ma and hence, a = 2.0 m/s2 . Thats the horizontal
acceleration.
where T is force due to spring
Since the crate only moves horizontally, the vertical
F ma = Ma
forces cancel, and therefore N = mg (as above).
or, F = Ma + ma
Therefore, the frictional force has magnitude
F
\ a= . f k = m k N = mk mg = m k (100kg )(10m / s2 )
M+m
Now, force acting on the block of mass m is = mk (1000N)
Newtons 2nd law, applied to the horizontal component
F = mF of the forces, gives us
ma = m .
M + m m + M
184

Fnet = ma Person is held up against gravity by static friction force


exerted on him by the wall.
fpush - f k = ma Acceleration of person is a rad directed in towards the
center.
500 N - (3.0 )(1000 N ) = (100 kg )( 2.0m / s 2 ) = 200N
108. (a) To find minimum ms we will take fs to have maximum
Solve for m k to get 0.30. value.
105. (d) Given m k , We can use Newtons 2nd law to figure out fs = msR; Sf y = ma y ; f s - mg = 0 ; m s R = mg ;
the creates acceleration during trial 3. Then we can
Sf x = ma x
use n = n 0 + at to find the velocity at time t = 1.0 s,
the moment the student stops pushing. mv 2
From Newtons 2nd law applied to trial 3, \R =
R
Fnet = ma
Fpush fk = ma Combining the equations

700N - (0.3)(1000N) = (100kg)a,


and hence, a = 4.0 m/s2 mv 2 Rg 2.5 9.8
ms = mg m s = = = 0.28 .
Since the crate speeds up by 4.0 m/s each second, its R v2 9.425
speed after 1 second is simply
109. (c,d) Mass of the person is cancelled as explained in above
v = v 0 + at = 0 + (4.0m / s2 )(1.0s) = 4.0m / s solution also smaller ms is larger should be the velocity
106. (c) Graph B would correctly show the crates velocity vs. to maintain in equilibrium
time. The crate speeds up while the student pushes it, ms= Rg / v2.
and then slows down while sliding freely across the
floor. Crucially, after the student stops pushing, the 110. (d) Work done in moving an object against gravitational
crate does not move backwards, as represented in graph force depends only on the initial and final position of
D. It continues moving forward, but at a slower and the object, not upon the path taken. But gravitational
slower rate. Therefore, after t = 1 s, the position vs time force on the body along the inclined plane is not same
graph continues upward, but with a smaller and smaller as that along the vertical and it varies with angle of
slope. when the crate stops, the graph levels off. inclination.
PASSAGE 2 111. (b) On a rainy day, the roads are wet. Wetting of roads
v2 lowers the coefficient of friction between the types
107. (d) arad = and the road. Therefore, grip on a road of car reduces
R
and thus chances of skidding increases.
112. (a) The force acting on the body of mass M are its weight
arad Mg acting vertically downward and air resistance F
acting vertically upward.
fs
R F
\ Acceration of the body , a = g -
M
Now M > m, therefore, the body with larger mass will
mg have great acceleration and it will reach the ground
first.

You might also like